39
Official LSAT PrepTest 73 September 2014 Form 4LSN110

Official LSAT PrepTestdocshare02.docshare.tips/files/27057/270577487.pdfLSAT® Writing Sample Topic Directions: The scenario presented below describes two choices, either one of which

  • Upload
    others

  • View
    2

  • Download
    0

Embed Size (px)

Citation preview

Page 1: Official LSAT PrepTestdocshare02.docshare.tips/files/27057/270577487.pdfLSAT® Writing Sample Topic Directions: The scenario presented below describes two choices, either one of which

OfficialLSAT PrepTest

73September 2014

Form 4LSN110

Page 2: Official LSAT PrepTestdocshare02.docshare.tips/files/27057/270577487.pdfLSAT® Writing Sample Topic Directions: The scenario presented below describes two choices, either one of which

© 2014 by Law School Admission Council, Inc.

All rights reserved. No part of this work, including information, data, or other portions of the work published in electronic form, may be reproduced or transmitted in any form or by any means, electronic or mechanical, including photocopying, recording, or by any information storage and retrieval system, without permission of the publisher. For information, write: Communications, Law School Admission Council, 662 Penn Street, Box 40, Newtown, PA, 18940-0040.

LSAT is a registered trademark of the Law School Admission Council, Inc.

All actual LSAT questions printed within this work are used with the permission of Law School Admission Council, Inc., Box 2000, Newtown, PA 18940, the copyright owner. LSAC does not review or endorse specific test preparation materials or services, and inclusion of licensed LSAT questions within this work does not imply the review or endorsement of LSAC.

2

Page 3: Official LSAT PrepTestdocshare02.docshare.tips/files/27057/270577487.pdfLSAT® Writing Sample Topic Directions: The scenario presented below describes two choices, either one of which

THE PREPTEST

3

• SECTION I .................................................................................. 4

• SECTION II ............................................................................... 12

• SECTION III .............................................................................. 20

• SECTION IV .............................................................................. 28

• ACKNOWLEDGMENTS .......................................................... 36

• COMPUTING YOUR SCORE .................................................. 37

• ANSWER KEY ........................................................................... 38

Page 4: Official LSAT PrepTestdocshare02.docshare.tips/files/27057/270577487.pdfLSAT® Writing Sample Topic Directions: The scenario presented below describes two choices, either one of which

1 -4- 1 1SECTION I

Time—35 minutes27 Questions

Directions: Each set of questions in this section is based on a single passage or a pair of passages. The questions are to beanswered on the basis of what is stated or implied in the passage or pair of passages. For some of the questions, more than oneof the choices could conceivably answer the question. However, you are to choose the best answer; that is, the response thatmost accurately and completely answers the question, and blacken the corresponding space on your answer sheet.

������� ������ �������� �� ��� �������� �� ��������� ������ �� ��������� �� ��� �������� �� ����������������. “������� ��������� ��� ���� ��� ���� ��� ������ ��������� ����� �� ������������,” �� ��������.

(�) �����������, � ����� �� ����-���������� ��������������������� ���������� ��� �������� ����� ������������ �� �������� ������’� ������ �� ���� �������. ����� ��������� ��� ��� ��������� �� ���������������� �� ������� ��� ���������� ���������� ����

(��) ������ ���� ������� ��������� �� ����������� ��� ����������� �� ����� �������’ ���� ��� ��������, ��� ������ ������� �� ������� �� ������� �� �������.

������� ��������� �� ��������� ���� �� ������ ������������, ��� ������� �� �� ��������’� ���� ���

(��) �������� �� �������� �� ������������� ���������� ��������� �������� ������������ �������. �� ��� ���������������������� ��� �����, ��� ����������� �� �������������� ��� ��� �������� �� ����� ������� ��� ��� ������ ����������. ��� �� ����, ������ �������� ��������

(��) �������� �� ���������� ���� ��� ��� ����������� ��������������� ������� ��� �� ������� ����� ������� ��������� ��� ������ �� �� ���� ����� �����������.

��� �������, ����� �� �� ���� ���� ���� ��������������� �� ������� �������� ������� ���������� ����

(��) ������� ������������ ������� ��� ��� ���� ������� ��������� ���������, ��� ������ ������� ������� �������������� ��� ������ ���, �� ��� ���� ���� ������������������ ���� ���� ��������� ���� ���� ������� ��������.�������� ��� �������� ���� �������, �����������

(��) ������� ������� �� � ����� ������ ��� ��� ��������� �����. ���� ������������� �� ��� ���� �� ��� ���������� ������ � ���������� ���� �� ������ �������������� �������. ����� ����������� ����� ��������� ���������� �� �������, ��� ����� ����������� ����

(��) ��� ���������� �� ��� ���� �� ��� ����������� ��������� ���������.

������������, ��� ����� �� ���� ����������� �������������� ��� ���������� ��� ������ �������������������� ���� ������� ��������� ��� ������� ��� �����

(��) �������’ ������� �� �������. ��� ���������� �� ������������ ���� �� ������� ����� ��� ��� �������������� �� ��� ������ �� �� ���������������� ����.������� ������ ������� ��� ��������� ������ ���� ��������� ��� ������������ ������� �� ������� ���� ��

(��) ����� ������ �� ������ ���� ���� �� ������, ���������� �� ��� ���������� ������ ���� ����� ������������ ���� ����� ��� ������� �� ��� ����� ����� ����� �������� ���������. ��� ����� ���� ���������������� ��� ��������� ���� ��� ��� ������� �����

(��) ���������� ����, �� ���� ��� ������� �� ��� ��������������������� ���� �� ��� �������� ������� ��� ���

������� �� �������. ��� ���� ���� ������� ��������������� ��� � �������� �� ���������� ���� ���������� �� ����� �� ��� ���� ��� ����������� �����,

(��) ��� ������� ����� ��� ���� ������� �� ������� ������������ ������ �� ��� ������. �� � �����, ������������ ��� ��� ������ �� ���� ����.

�. ����� ��� �� ��� ��������� ���� ���������� ��������� ��� ���� ����� �� ��� ��������

(�) �������� ���� ��� ����� �� ������� ���������� ��� ������ ��������������� ����� ���� ������� ��������� �� ��� ���� ������� ����� ������ ���������.

(�) ��������� �� ������ ��������������� ����������, ��� �������� �������� ���� ������� ��������� �� ����������� ��� ��� ������� �� ���� ������� �������.

(�) ��� �������� ������������ ���� ������� ��������� ��� ������� ����������� �� ���� �� �������� �������.

(�) ������ ��������������� ��������� �� ������ �������� ���� ������� ��������� �� ����������� ��� ��� ������������ ������.

(�) �������� ���� ��� ����� �� ���������� �������� ����� �� �������� ��� ����� ���� ������� ��������� ������� �� ��� �������� �� ��� ������� �������.

�. ��������� �� ��� ������, ������� ���� ���� �� ������� ������������ ������������� ������� ���� �������� ������� �� ������� ����� ��� ������� ����

(�) ��� ������� ���������� �� ������� �������(�) ���� ���������� �����(�) ���� ������ �����������(�) ����� �� � ���� ����� �� ������������(�) ���� ���� �� ��������� �������

GO ON TO THE NEXT PAGE.

73

Page 5: Official LSAT PrepTestdocshare02.docshare.tips/files/27057/270577487.pdfLSAT® Writing Sample Topic Directions: The scenario presented below describes two choices, either one of which

1 1 -5- 1 �. ��� ������ ������� ����� ��� �� ��� ��������� ���������

��������� �� ������� ���������

(�) ��� �������� �� ���� ��������� ��� ��� ������ �� �� ���������� �����������.

(�) ��� �������� �� ���� ��������� ����� ������ ������� ���� ���� ����������� ���� �����.

(�) ��� �������� �� ���� ��������� ������ �������’ �������� ������ ���� ����� ����������.

(�) ��� �������� �� ���� ��������� ���� �� ������ �� ������������ �������.

(�) ��� �������� �� ���� ��������� ����� �� ������ ������ ���� ������� �������.

�. ��� ������ ����� �� ���� ������ �� ����� ���� ����� ��� �� ��� ��������� �����������

(�) ������� ��������� �� ����������� ��� ������ ���� �� ��� ��������������� �� �������� �������.

(�) ��� ���� ���� � ������� ���������� �� � ������� ����������� �� ��� ����� �� ��� ���������� �� ���� �����������.

(�) ���� ������������ ������� ���� ������� ���� ��������� �� � ������� ��� ����������� �� ���������� �����������.

(�) ����� ������ ������� ��� ��������� ������ �� ������� �� ����� ������������� ����������.

(�) ������� ��������� �� ������ ��� ���������� ��� ���� ��� ��� ��� �������� �� ���� �������.

�. ��� ������’� ������ ������ ��� ��������� �� ��� ������ ��������������� ���������� ��� ���� ���������� �� ��������� �� ��� ��

(�) �������� ������������(�) ���� �����������(�) ����-������ ����������(�) ����������� ���������(�) ���������� �����������

�. ����� ��� �� ��� ��������� ���� ���������� ��� ���������� ��������� ��� �������� �� ��� ������ ��������� �� ��� ��������

(�) �� �������� ��� ���������� �� ����������� ������������ ������ ������ �� ��� ������ ���������������� ��������� �� ��� ����� ���������.

(�) �� ����� ������ �������� ���������� ���� ��� ������ ��������������� ������ ��������� �� ��� ����� ��������� ��� ���������.

(�) �� ��������� ��� ������ ����������������’ ���� �� ������������ ������ �� ����� �� ������� ��� ��� �������� ��������� �� ���������� ���������� ��� �������� ������ �� ���� ���������.

(�) �� ���������� ��� ��������� ��� ��� ������ ��������������� �������� ���� ��� �������� �� ��� ���������� ���� ������.

(�) �� �������� ��� ������������� �� ��� ������ ����������������’ ������ ��� ����� ������� ��� ��������� �� �������� ������� �� ���������� ����������.

�. ��� ������� ������� �� ��� ������� �� ��

(�) ����� �� ����� �� � �������� �������� ����������(�) ��������� � ������������ ������(�) ����������� ���� � ���������� ���� �� ���������(�) ��������� ��� ���������� �� � ����������� ������(�) ������� ��� � ���������� ������ �� �������

����������

GO ON TO THE NEXT PAGE.

73

Page 6: Official LSAT PrepTestdocshare02.docshare.tips/files/27057/270577487.pdfLSAT® Writing Sample Topic Directions: The scenario presented below describes two choices, either one of which

1 -6- 1 1���� � �������� ���������� �� ��� ���� �� ��������������������� ����� �������� �������.

���� ������� ������ ��� “�����-�������”��������—����������� �� ����� ��� �� ���� ��������������� �������, ����� �������’� ���������, ���������� ��� �����, ���������, �����������, �� ��������—

(�) ���� ������������ ������ �� ��� ����� ������� ��������������� ����� ���� ���� �����. �� ���� �������� �������� ���� ���������� �� ��� ������ ����� �������� �������������� ��������� ��� ���� ���� ������ ����������� ��� ���������� ����� ������� ���� ���� �� �� ���

(��) ������������ �� ������� ��������. �� ������� ��������� ����� ��� �� ������ ������� ����� ����� ���������� ������� ���� �����, �� ��� ��� ������� �� ��������� ���� ����� ������. ����� ������, �� ������, ������� ���� ��� ����������� ����� ���� ��� �����. ��

(��) ������� ��� ��������� �� ��� ������� �� �������������� ����� �� ������������ ���, ��� ���� ����� ������� ��� ����������� �� ��������� �����������—��������� ������� ���������’� ������������� �������� ��� ���� �� ����—������ ���� ����� ��� ����

(��) ����� ������.�� �� ��������� ��� ������’� �������—��� ��������

��������� ���� ��� ������� �� ������—���� ��� ������������’� ������������� ��� ������������� ��� ������������ �����. �� �� ��� ����� �� ��� �������, ������ ���� ���

(��) ������� ����� ��� ��� �������� �� ��� �� ��� ��, ��������� ��� �� ����� ��������� ��� �������, ���-�����-��-����� �����������. ���� ��� ���� ���� ���� ���������� ���������� ��� ������ ��� �������� ��� ��������������� ��� ��� ������� �� �� ���� �������� ���

(��) ������ ������� ��� ���� ��� �������� ��� ��������� ������������� ��� ������ ����������� ���� �� ��� �����.��� ��� ���� ������ ������� ��� �� ��� ������ �� ������������� ������ �� ��� ����������, ��� “����” �� �������. ���� �� ���� �� � ��������� �������� �� ���

(��) ������� ��� ���������� ���� �� ���� �� � ������������������� �� ������. �� ��� ������ ����� �� �������������’� ����������—�� ���� ������’� ������������ ���� ��������. ������� ��� ��������� ��� ����������,��� ���� �� ������� (��� �� ����� ���� �� ��� ����) ����

(��) �� ��� ������ ���� ���� �� �����. ����� ������������� ���������� ������ ��� ����� ������ ����� ���������� ������.

���� ����� �������’� �������� �� ������ ������������ �������—����� ������ �� ��������� �� �����������

(��) �� �� ������������ �������� ����—�� ����� ������������������� �� ���������� ��� ��������. ����� ������� �� ������, ��� �������, ��� ���� ��� ����� ��� ��������� ���� ������������ � ����� ����� ������������ ����������� ��� ��� ����� �� ����� ������

(��) �������. ��� ����� ������� ��� �������������. ��� ����,��� ������ ����� �� ��� ������� ����� ����� �� ����� ����� �������� ������� �� ��� ��������. ��� �������, ������ �����, �� ������� ��� ����������. ������������’� �����-������� �������� ���� ���� ��������

(��) �� ���� ������� �����������, ��� ������� �� ���������� ��� �� ���� �� ���� ������� ����������� ��� �������, ��� ������� ���� ��������� �������.

�. ����� ��� �� ��� ��������� ���� ���������� ��������� ��� ���� ����� �� ��� ��������

(�) ��� ������������� ����� ����� �������’� �����-������� �������� ���� ����� ������ ���� ��������������� �������.

(�) ��� �������� ����� �� �������’� �����-������� �������� ������� ���� ��� ������’� ������������ ��������� �� ��� ��������� ����� ��������� ��� ��� ������������� �� ��� ���������.

(�) ��� �������� ����� �� �������’� �����-������� �������� �� ���������� ���� ��� ������������ �� ������� �������� �� ���������� �� ��� ������������� �� ��� �������.

(�) ��� ���� ���������� �� �������’� �����-������� �������� ���� �� ��������� ����� �� ���� ��� ����� �� ����� ��� ������ �� ���������� ������� ���� ��� ������� ��� ������� �� ���� �������.

(�) ��� �������� �� �������’� �����-������� �������� �������� �� ���� ���� ���� �� ����� ��� ������� ��� ��� �� ��� ��������� ������ ���� �������.

�. ��� ������ �������� ��� ����� �������� �� ��� ������� �� ������ ��

(�) �������� �� ���������� ������� �� ��� ����(�) �������� �� ��� �������������� ����� �� �������(�) ����������� �� �������’� ���������(�) ����������� ���� ��� ���� �� �������� ����������(�) ������� ��� � �������� ��������� �� ��� ����

��. ����� ��� �� ��� ���������, �� ����, ����� ���� ���� �� ������� ��� ����� ����� ���������� �� ��������� �� ����� ��–���

(�) ������� ��� � �������� ��������� ����� ���� ������ ���� ������� ��� � ����������.

(�) ���������, ������ �����������, ��� ������ ��������� ���������� ����������.

(�) ��� �� ��� ������� ��� � �������� �� ��� ���� �� �� ������� �� ��� ���� ����.

(�) � ������� ��� �������� ������� ����� � ������ ���� ��� �� ���� ���� �� ��������� �������.

(�) ��������� ��������� ���� ��� ��������� ������� �� �� ������, ������, �� ������.

GO ON TO THE NEXT PAGE.

73

Page 7: Official LSAT PrepTestdocshare02.docshare.tips/files/27057/270577487.pdfLSAT® Writing Sample Topic Directions: The scenario presented below describes two choices, either one of which

1 1 -7- 1 ��. ����� �� ��� �������, ������� �� ���� ���� ����� ��� ��

��� ��������� �� �������� �� ��� �����-������� ���������

(�) � ���������� ��� ���������� ����������� �������� �� �������� �� ��������� �������� ������� ���������� ��� ��������

(�) � ��� ������ ����� ������ ��� �������� �� ������� ��� ������� �� � ���� ������� �� ��� ���������

(�) � �������� ����� ����� ������� � ������� �������� ���� ������ ���� ��� ������� ����������� ��� �� �������� �������

(�) �� ��������� ����� ��������� ��� �������� �� �� �� ���������� �� ��������

(�) � ���� �������� ��� ������� �������� ������ �� ������ �� ����� �� ���� ��� ���������� �� � �����������

��. ����� �� ��� �������, ��� ������ ����� ����� ���� ���� �� ��� ��������� ���������� �������

(�) � ���� ��������� ������ ��� �� ���� ��������� �� ���������� �� ��������� ���� � ���� ��������� ������.

(�) �������������� �� � �������� ������� �� ���� ����� �� ���.

(�) ���� ����� ������ �� �� �� ����������� �� � ��������� ���������� ��� �������� ������� ��� ��������� �����.

(�) �� ��� ��������� ����� �� ���� ��� ���� ��� ��� ��������� ������� �� �� ����� �� � �����.

(�) � ���� �� ��� �������� ���� �� ��� ������ ���� �� �������� ��� ������’� ����������.

��. ��� ������� �������� ��� ���� ������� ��� ��������� ���� �� �������’� ���

(�) ����� ��� ������ �������� �� ����������� ����������� ������������ ����

(�) ����������� ��� ��������� ������ �� ������� ��������

(�) ��������� ������ �� ������ ���� ������ ���� �����(�) ����� ���� �� ������������ ������’� ������(�) ��� ���� �������� �� ���� � ������� ��� �����������

��. ��� ���������� �� ���������� �� ��������� �� ��� ������ ��������� ������ ����� ��� �� ��� ��������� ���������

(�) �� �� ��� ���� ���������� �� ��� �������, ��� ����� ��� ���������� �� �������’� �����-������� �������� ������ �� � ���� �����.

(�) �� ���������� � �������� ��� ������ ���� �� �������������� ��� �������� ������ �� ��� �������� �� �������’� �����-������� ��������.

(�) �� �� ��� ��� ���� �� �� �������� ���������� ��� ������’� �������� ��������� �� ��� ������� �� ��������� �����������.

(�) �� �� ���� �� ������� � ��������� �� �������’� �����-������� �������� ���� ��� ������ ����� �� �� ������������ ��������.

(�) �� ����� � �������� ������� ��������� �������� ��� ����� �� ������� ��� ������’� ���������� ���� �������’� �����-������� �������� ��� ���� ���� ��� ������.

��. ��� ���� ������� �� ��� ������� ��

(�) �� ��������� �������’� �������� ����������� �� � ������������, ����� ���������� �� ��� ����������� ��� ������� �� ������

(�) �� ����� ���� ��� ������� ������� �������’� ���� ��� ��� ������� ��� �������� �� ���� �� ��� ����� �������� ��� �����’ ��������� �����

(�) �� ���� ���� �������’� ����������� ���������� ������ �������� ��� ���� ��� ��������� ��� ��� ���������� �� ��� ������������ ������

(�) �� ������� ��� �������’� ������� �� ��������� ��� ����������� ��� �������� �������� �� �������� ��� ������ �� �������

(�) �� ������ �������’� ����������� ��� ������� �� ������ ������� ��� ���������� �� ������� ���� �� ���������� ��� ������ ������� �� ��� �������

GO ON TO THE NEXT PAGE.

73

Page 8: Official LSAT PrepTestdocshare02.docshare.tips/files/27057/270577487.pdfLSAT® Writing Sample Topic Directions: The scenario presented below describes two choices, either one of which

1 -8- 1 1���� ������� �� ����������� ���� ������� ���� ���

�������� �� ����� ����� �� ��������� �� ��� ������������������ ���������� ���� ����� ������� ������ �� ��������������� ��� ��������� �����. ������� �� ����

(�) ���� �� �������� ��� ��� �������� �� ��������� ��������������� �������, ��� ���������� ���� ������ ������������� �� ���������� �� ��������� ������������������� �� ����� �� ����� ������ �� �� ����������������������� �����. ���� ������� ������ ������� ��

(��) ���� ������ ������� ��� �������� ����������������������� �� ����������� ������ “�����” ���� ��� �������� ����� ������������ ���� �����������, ��� �� ���������� ����-����� �� ���������, ��� ������ �� ���������—��� ������� ��� ���������������� �����—

(��) �� ������������.������� �������� ���� �� ��� ���� ������� ����

�����, ���� �������� ��� �������������. ����������������������� ����� ����� ��� ��� ��� ��������, �������������������� ������������ ������� ���� ���

(��) �������� �����, �.�., ������� ��� ��� �������,������� �������� � ����� “����” ��� ����� �����.����� ��� ����� ��� ����������� �� ���� ��������� ������� ���� ������� �� �������� �����, ��� ���������������� �� �������������� ��� ����� ������ ���������

(��) ��� ��� �������� ������� �� ���� ����� �����������.�������������, ��� ����������� ������� ���� ���

����� ����� ���� ����� ���� �������� ������� ����������� �����������. �� ���������� ��� �����, ��������, ���� ��� ���������, �������� ��� ���� �����

(��) ���� ��� ������� ����� ����� �� ���� �� � ������ �� ��������������� �� �����������. ���, �� ����� �� �� ��, ������� �� ��������� �� ��������� ������ �� �������������� ��� �� ��������� �� ������� ���� ���� ���� ���� �������� ��� ����������� ��������� ����� ������.

(��) ���, �� ����, ���������� ���� � ����� ������� ���������� ���� ��� �������� �� ��������� ��� ��������� ����������� �� ��� �� �� ������������. ��������������������� ��� ������ �� ������ ��������� �������� ����������, �������� ������, ��� ����������� ��������

(��) �������������� �� ����������� ������. ��������,�������� �������� ���� ���� ������ ���������� ������������ ��� ���������� ���� ��� ��� ��� ������������� �����������. �� ����� �� � ���� ���� ������������ �����������, ��� �� �� ��� ������ ���

(��) ������������� ������ ��� ����� �� ��������� ��, ��������, �������� ����������� ��� ������ �� ������ ��������� ��� �������� �� ����������� �� ���� ���������� ��������� ������������ �������� �� ������������������������. �� �� �� ����� ���� ���� �� ����—�������

(��) ���� ����—����� ��� ��� �� �� ���������� ����������� ��� ����� ��� ������� ���� �� ��������� ������������ �������������� ���� �� �������. ���� ���� �������, �������, ���� ��������� �� ��� ������ ���������������� ��� ��� ������� �� � ����� �� ������� ����

(��) �� �����������, �� ���� ���� ��� ������� ����������� ������� ���� �� ���� ���� ��� ������������� ��������.

��. ����� ��� �� ��� ��������� ���� ���������� ��������� ��� ���� ����� �� ��� ��������

(�) ����������� ��� ������� ������ ����� ���� ���������� ���� ��������, ������� �� �� ���� �� ��������� ����� �� ��������� ��������� ��� ����� �� ������������� ������������ ������ �� ���� �������������.

(�) ���� ��, �� ���������� ���� ������, ����� �� � ����������� ���������� ������� ���� ��� ����� �����, ���� ���������� �� �������� �� �������� �� ��� ������������ �� ��������� �� ��� ������ �� ���������� �� ������-���������, ��������-�������� ���������.

(�) ��������� ��������� ��������� �����������’ �������� �� ����� ����� ��� ��������, ������� ����������� ��� ���� �� ���� ���������� ��������� ��������� ����� ����� ��� ��� ���� ���� �� ��� ��� �������� �� ���� ������ ������� �� ������� ������� �����������.

(�) ������� �� ����������� ��������� ����� �� ��� ����������� �� ����� ����� �� ��� �������� ��� �� ��� ����� �������� ��� ������� �� ������ �� ���� ����������� ������� �� �������������� ����� ��� ���� ����� ���� ��� �������� ����� ���� ����������� �������.

(�) ��� ����������� ����������� ���� ���������� ���� ����� ������� ���� ��� ����� ����� �������� �� ���������� ����� ��� ����� ������� �� �����������, ������� ��� ����������� ��������� ���������’ �������� ��� ������������� �����.

��. ��� ������ ������ ���� ������� �������� ���� �����������

(�) ���� ���� �� ����� ������������ ���������� �� ������������� �������� ��������

(�) ������ �� ������’� ���� ����� �� ����� �� ������ ����� �����

(�) ��� ���������� �� � ������ �� ����������� ����������� ��������������

(�) ���������� ��� ��������’� ���� ��� ����������� ��������-������

(�) ���� ���� ��� ����� ���� ������ �� ������ ��� ���� ���� ���� ����� �����

GO ON TO THE NEXT PAGE.

73

Page 9: Official LSAT PrepTestdocshare02.docshare.tips/files/27057/270577487.pdfLSAT® Writing Sample Topic Directions: The scenario presented below describes two choices, either one of which

1 1 -9- 1 ��. ��� ���� �������� �� ��� ����� ��������� �� ��

(�) ��������� ��� ��������� ��� �������� ������� ���� ����� ��������� ��������� �� ����������� �����

(�) ������� ��� ���������� �� ����� ����� ����� ��������� �� ���� ������ �������

(�) �������� ��� ������������� ��������� �� ����� ��� ������������ ���������� �� ���� ������ ����������� ��������� �����������

(�) �������� ��� ���������� ����� ����� ������������ ������� �� ����������� ��� ���������� �������’� �������� �� �������� �� ����� �����

(�) �������� ��������� ����� ��������� ���� ������ ������������ ��� �������� ����� ���� �� ������� ���������� �� �����������

��. ����� ��� �� ��� ��������� �� � ����� ���� ��� ������ ���������� �� ��������

(�) �� ������ �������, ����������� ����� ���� ������ �� ����� ���� ���� ��� ���������.

(�) ������ ��������� ������ ���� ������� ��������� �� ���� ���� ���� �� ������� ����� �������� �����.

(�) �� �� ���������� �� ���� ��� ���������� ����������� ������� ���� ��� ����� ������������� ����� �� ������ �������.

(�) ����������� �� ������ ������� ��� ��������� ������ � ���� �� ���������� ������� �� ��� ������� �� ������������ ��������� �������.

(�) ����������� �������� ����� ����� ����� �� �������� ���� ���� ������� ��������� ����� �����, ������ ���� ������ ���� �����, ������ ���� �� ��� ��������� �� ���������.

��. �� ��� ���� “������ �� ����������” (���� ��), ��� ������ ���� �������� ������ ��

(�) ������������� ��������� ������ ���� ���� ��������� ����� ��� ������ �� �����������

(�) ������, ����������� ������ ���� ���� ��������� ��� ��� ����������� �� ����� ��������

(�) ��������� ��������� ���� ���� ��������� ����� ��� ������� ���� ����������� �� ������� �� � �����

(�) ������ ��������� �� ������ �������������� ���� ���� ��������� ��� ��� ��������� �� ��� �����

(�) ������������ ���������� ���� ���� ��������� ��� �� ������������ �� ����� �������� �� ����

��. ����� ��� �� ��� ��������� ��������� ����� ���� ��������� �������� ��� ��������

(�) ���������, ����� �� ��������� ����� ����� �� ������� ��� ������� ���� ����������� �� ����������� �� �������, ��� ��������� �������� ���� ��� ������� ���� �������.

(�) ���������, �������� ��������� ������ ����� ����������� ��� ���������� ���������, ��� ������� �� ���� ������ ������� �� ����������� �� �� ����� ��� �� ����� ������ ��� ��������� ����.

(�) ���������, ��� ����� �� ����� ����������� �������� �� ������� ��� ������ �� ����������� ���� ������� � ������ �� ��� ���������� �� ����� ������ ���� �� ��������� �������.

(�) ���������, ����� ����������� ���� ����������� ������ ������� �� �����������, ���������� ���� ������ �� �������� ���� ���� ������� ��� ������� ���������� �� �������� ������ ��������.

(�) ���������, ��� ��������� �������� �� ����������� �� �������� ������ �� ��� ����� ���� ����������� ����� �������� ����� ���� ����� ��� ������ ��� ��� ������ �� ��������� �� ����������� ���� ���� ����� �����.

GO ON TO THE NEXT PAGE.

73

Page 10: Official LSAT PrepTestdocshare02.docshare.tips/files/27057/270577487.pdfLSAT® Writing Sample Topic Directions: The scenario presented below describes two choices, either one of which

1 -10- 1 1Passage A

����� ��� ��� ���������� ���� ��� ����������� ��� ������ �� ������� ��������� ��������. ��� ����������� ������� �� ����������� ��������� ��� ���������� ���������� ������� ��� ������������ ���� �� ���

(�) ��������� ���� ��� ���������� ��� ����� �� ������.��� ��������� �� ������� �� �������� ��������� ������������� ����� ����� ��� �������� �� �������� ������� ������ �� ������� �� ���������.

����� ���� ����������, �� ��� ����� ���� ������(��) ����, ��� ��������� ���������� ����� ������������ �����

��� ������� �� ������� ��������� ����������. � ������ ��� �������� �������� �� ����������

���� ��� ��������� �� ������� �� ����������� �� ���������� ���� ��������.

(��) �. � ������ ��� �������� �������� �� �������������� ��� ��������� �� ������� �� ��������, ���� ����������� ��� �� �������� �� ��� ��������, �� �������� �� ���

��������.�. �� ��� �� �������� �� ��� �������� ������ ��

(��) (��������) ������������ �� � ��� �.�������, ��� ��� ������ ���������� ��� ��������� ��

���������� ���� ��� ���������� �� ������� �� �������������� ������� �� ��������. ���� ������ ����� ���� �������� ������� ����, ��� �������. ��� ��������� �� ����

(��) ��������� ������ ��� ����� �� ��� ������������� �� ���������.�� ���� ��������� ��� ������ ������� ��������� ��������� ����, ����, �� ��������, ����� �� �� ���� ��������� ���� ���������� � ��������� �� ������������������ ��� ���������� ����������� ����� ��������

(��) ���������� ��� ���������� ���� �� ����, ��� ���������������� ��� ������ ������ �� ������ ���� ������ ��������� ����������, �� ������� � ����������� �� ����������� ��������� ���� ������ ���� ��������.������ ��������� �� �������� ���� ���� �� �������

(��) ���� ���������� ���� ���� �����������. Passage B

�� ����, ��� ������ ������ �������� ������ ��������� �������������� ���, ����� �������� ���� ������������ �� ����� ���� ������ ��������� �� ������ ���������� �� ��� ������� ����������. ��� ��� ��� ���

(��) ���� ������� �� ��� �������� �������, ��� �� ��������� ������ �����. ��� ������� �� �����. �� ��� ����� ����������� �������� �� ������ ��������� ����������������� ����������� �� ������ �� ��� ���������������’ ���� ��������. ������� ����� ���� ����

(��) ��������� �� ������ �������� ������ ��� �������� ������� ���� �� ���� ���� ��� �������������� ������� ������.

��� ������� (��� ����� ������ ��� �������) ����� ��������� ����� ������ �������� ������ �� ���� ��

(��) ����� ������� �� ����� ������ ��������� ���� ��� ���������� ��������� �� ���� ����. ������ ��� ���������������� �� ����� �������, ��� ���� �������� �� ����.�� ��� ������ �� ���� �������� ��� ��� ���������, ������� ��� ��������� ����� ���� ����. ��� ������� ������

(��) ���� � ����-������� ����� �� ��� ����, ����� ��� ������������� �� ����� �����. �������, ��� ���� ������ ���������� �� ��� �������� ������. ���� ��� �� ����������������������� ����� ���� �� �� ����. ��� ��� ������������� ��� ���� ������ �� ������� �� ��������� ��� ����

(��) �� ����—�� �� ��������� �� �������� ���� �� ��������.

��. ����� ��� �� ��� ��������� ���� ���������� ��������� ��� ���� ������� ��� ����� ������� � ��� ������� ��� ��� ���� ������� ��� ����� ������� � ��� ��������

(�) ������� �� �� ������� � �������� �� � ����� �������

������� �� �� ��������� � �������� �������� �� � ����� �������

(�) ������� �� �� ������ � ������� ������� �� � ������ �� ����� ������

������� �� �� ���� � ���������� ����� �������� �� � ���� ����

(�) ������� �� �� ����� ��� ��� ������� �� ��� ����������� ����������

������� �� �� ������� � ���� ���� ����������� � ����� �����

(�) ������� �� �� ����� ��� � ���������� ����� ������������ �� �� �������� ��� ����������� �� � ����� ������

(�) ������� �� �� �������� ��� ��� �� ������� ����� ����������

������� �� �� ������� � �������������� �� ���� ������ ���� ����� ����������

��. ���� �������� ���������� ������� ����� ��� �� ��� ����������

(�) �������� �� �������� ���� ��� ����� �� �������(�) � ����� ����� ��� �������� �� ��������(�) ����������� �� �������� �� � ������ ���� �����(�) �������������� �� ������������� �� ���� ���������(�) ��������� ��������� �� ���� �� �� ��������

��. ����� ��� �� ��� ��������� �� ���� �� ��� ������������ ������� ������� � ��� ��� ������ ��������� �� ������� ��

(�) ��� ������ ��������� �� ������� � �������� �� ������� � ������� ������� �� ��� ������ ��������� �� ������� �.

(�) ��� ������ ��������� �� ������� � �������� �� ����� ����� ���� ������� ��� �������� ���� �� ������� �.

(�) ��� �������� �� ��� ������ ��������� �� ������� � �� ������������ �������� �� ��� �������� �� ������� �, ��� ��� ������� ������ �� ��� ��� �� ���������.

(�) ������� � �������� � ������ ���� ����� �� ������� ��� �������� ��������� �� ��� ������ ��������� �� ������� �.

(�) ��� ������ ��������� �� ������� � �������� �� ��������� ��� ������ ��������� �� ������� �.

GO ON TO THE NEXT PAGE.

73

Page 11: Official LSAT PrepTestdocshare02.docshare.tips/files/27057/270577487.pdfLSAT® Writing Sample Topic Directions: The scenario presented below describes two choices, either one of which

1 1 -11- 1 ��. ����� �� ���� ��� �� �������� ���� ����� ������, ���

������������ ������� ����� ��� �� ��� ��������� ����� �� ��������� �� ���� ��������� �� ��� ������������ ������� ������� � ��� ������� ��

(�) “���� �������� ��� ��������� � ���’�-���� ������ ��� ������� ��� ������” “��� ����� �� ��������”

(�) “����� ��������� �� ����� �� �����” “��� ����� ��������� ������ �� ���������”

(�) “������� ����� ������ � ����� ��� ��� ������” “���� ������� ��� ��������� �����”

(�) “������� �������� �� ��� ����� ������� ����������� �����” “��������������’ ���� ���� ���� ��� ����� ����������� �����”

(�) “������������ �� �������� ������������ ��� ������” “����������� ������� ��� ������� ������ �� ��� ������ ��������”

��. ��� ������ �� ������� � ����� �� ���� ������ �� ������������ ��� ������� �� ��� ������ �������������� ��� �� ����� ��� �� ��� ����������

(�) �������������� �� ������ �������� �������� ������ ��� ���������� �������

(�) ������������� �� �������� ���� ��������� �������� �� ��������

(�) ��������� �� ���������� �� ��� ��������� �� ������� �� �����������

(�) ���������� �� ���������� �� ��� ��������� �� ������� �� ��������

(�) �������������� �� � ��������� �� �������������

��. ����� ��� �� ��� ��������� ���� ���������� ��������� ��� ���������� �� �������� ����� �� ������� � �� �������� �� ������� ��

(�) ������� � �������� � ������� ���� ������� ��������� �������, ����� ������� � �������� �� �������� ���� �� ���� ��� ����������� �������.

(�) ������� � ������ ��� ��� ����������� �� ��� ���� ���� ��������� �����, ����� ������� � ��������� ���� � ������ ����.

(�) ������� � ������� �������� ���� ���������� �� ������� � ������ ��������������, ����� ������� � ������ �� ��� ����� �� ����������� ����������� �� ������� ��� ������.

(�) ������� � ������� ������ � ���� ��� ���� �������� �� �������� ��� ��, ����� ������� � �������� � �������� ��������� �� � ���� ��� �� ��������.

(�) ������� � �������� �� �������� �� ������� �� � ����, ����� ������� � �������� �� ��������� � ����.

S T O PIF YOU FINISH BEFORE TIME IS CALLED, YOU MAY CHECK YOUR WORK ON THIS SECTION ONLY.

DO NOT WORK ON ANY OTHER SECTION IN THE TEST.

73

Page 12: Official LSAT PrepTestdocshare02.docshare.tips/files/27057/270577487.pdfLSAT® Writing Sample Topic Directions: The scenario presented below describes two choices, either one of which

2 -12- 2 2 2SECTION II

Time—35 minutes25 Questions

Directions: The questions in this section are based on the reasoning contained in brief statements or passages. For some questions, more than one of the choices could conceivably answer the question. However, you are to choose the best answer; that is, the response that most accurately and completely answers the question. You should not make assumptions that are bycommonsense standards implausible, superfluous, or incompatible with the passage. After you have chosen the best answer, blacken the corresponding space on your answer sheet.

�. ���������� ��� ���� ��� ������ � ���������� �� ������� ��� ������� ������� �� ������ ���������. ���� �� ������� �� ��� ����������� �������� �� ���� ���������� ��� ��������� �� ��� ������� ����������� �����������. �� ��� ���� �������� � ���������� �� ������� �� ����� ����������� ��� �����������, ����� �� �� �������.

����� ��� �� ��� ��������� �� �� ���������� �������� �� ��� �������� �� ��� ����������

(�) ��������� ����������� ������� ������ ��� ���� ����������� �����������.

(�) ����� ��� �� ����������� ���� ����� ���� �� ������� �� ����� ����������� ���������.

(�) ����������� ��� ���� ������������� ��� ��� ��������� �����������.

(�) ��������� ����������� ��������� ��� ������� ������� �� �� ��������.

(�) ��� ���������� ����� �� ��� ���� ��� �������� ���� �� ���� ���������.

�. ������� ������������, �’�� ������� ���� ���� � ����������� ������ � �������� ��� ������ � ��������, ��� �������� ���� ���� “����� ���” ������� �� ������ “���’�� �������.” �’�� ���� ������� ����� ���� ������. ��� ���� � ������ ������ � ������ ��� � �����, ��� �������� �� ������ “���’�� �������.”

����� ��� �� ��� ���������, �� ����, ���� ����� �� ������� ��� ����������� ���� ������ �������� �� ������’� ����������

(�) ��������� ������ ���������� �� ����� ����������� � ����� �� ������ ���� ���� �� ������� �� ������� ����.

(�) ����������� ��� ����� ���������� �� ����� ��������� �� ����� ���������, ������� ��������� ��� ���� �� ��� ���� ���� ����.

(�) ����������� �� ��� ������ ��������� ��� ��� ���� ���� �� ����� ���� � �����.

(�) ��� ��� ���� ������ ������� �� ����� ������� �� ��������� ���������� �� ����� ������ ���� �� ��������� ��������.

(�) �� � ���������� �����������, �� ������� �� � �����, ��� �������� ����� ���� ��� �������� ��� ������.

�. ���� ����� ���� ������ ���� ���� ��� ����� ������ ��� ������� �����. �������, ���� ���� �� ������ ���� ���� � �������� �����������. ����� ���, ���������� ���� ��� ����� ������ �� ��� ������� ���� ��������, ��� ��� ������ ���� ���� ���������� �� ��� ���� ��� ����� �� ������� ��� ��� ������ �����. ���������� �������� �� ��� �������� ����� ����, �������� ������ ���� ��� ��������, ���� ������.

��� ��������� �� ��� �������� �� ���� ���������� �� ��������� �� ���� ��� ��������

(�) ����� � ������� ���������� �� ��� ����� �� ���� ��� ���������� ����

(�) ������ ���� � ������� ���� �� �������� �� ��� ������ ������ ���� ��� ����� ���� ��� ������� ��� �������� �� �������

(�) �������� �� � ���������� � ����� ������� ��������� �� �������� ��� ���� ����������

(�) ����� ��� ������� ���� �������� ���� ������� ������� ���� ��� �� ��������� ����� ���� �� �� ������� ����� ����������

(�) ������ � ����������� ��� � ������� �� �� ������� �� ��������� ���� ������� ���� � ������� ���� �� �������

GO ON TO THE NEXT PAGE.

73

Page 13: Official LSAT PrepTestdocshare02.docshare.tips/files/27057/270577487.pdfLSAT® Writing Sample Topic Directions: The scenario presented below describes two choices, either one of which

2 2 2 -13- 2�. ���������� ��� ��������� �� � ����� ������� �����

������������ ������� �� ���������� �� ������ �� �� �������� ���������� �� ������ �� �� ��������������� �� ��� ���������� ���� ��� �������� ��������� ���� ��� ������� ��� ��������� �������.

����� ��� �� ��� ��������� ��������� �������� ���� ������� �� ��� ��������� ������ ������

(�) ��� ��������� �� ��� ������� ����������� �� ���������� ��������, ����� �� �������� �� ������ ��� �������� ������ �� ����� ���� ���� ��� ������� �������� �� ������� ��������.

(�) ��� ��������� �� ��� ������� ����������� �� �������� ��������, ����� ��� ����� ������ ������������ ������� ��� ���������� ���� ����� ���, ���� ��� �������’� ������� ���� �� �� ���-���� ����.

(�) ��� ��������� �� ��� ������� ����������� �� �������� ��� ��������, ����� ��� ����� ������������ ������� ��� ���������� �� ��� ������� ����� �� ��������� ������� ��������� ��� �������� �� �� �������� ������������ ����������.

(�) ��� ��������� �� ������� ����������� �� �������� ��������, ����� ��� ������� ����� �� ��������� ���������� ��� ������������ �� ��������� ��� ������ �� �� �������� ���������� ��� ��� ���� ����� ��������� ���� �������.

(�) ��� ��������� �� ������� ����������� �� ���������� ��� ��������, ����� ��� �������� ���������� ��� ����� �� ��������� �������� �� ������ �� ��������� �������� ��� �� ����� ��������� ���� �������.

�. ������� ������� ����î��� ������ ���� ��� �������� ����� ���� ��� ��������� �� � “�������� ����,” � �������� ����� �� �������� ������� �� ����� ��� ����. �� ���� �� �������, ��� ������� ������������ ������ ������ �������� ������������ ���� ���� ��� �������. ���� �� ��������� ���� �� �������. ��� ������� ����� �� ������� ������—��� ����������� �������� ������—���� ����� ������� ���� ���� ����������, ����î���’� ������ ���� �� ���������� ����������.

����� ��� �� ��� ��������� ���� ���������� ��������� � ���� �� ��� ������� ������’� ����������

(�) ��� ���������� �� ������� ������ ���� ���������� ���������� �� � ��������� ������ ����� ����������� �� ��� �����������.

(�) ��� ���������� �� ����� �� � ����� �� ������� �� � ��� ���� ���� ��� ���� �� ��� �������� �� ������� ����.

(�) ��� ������� ������ ����� ��� ������� ��� ��������� �� � ������ ���������� ������� �������� ���������.

(�) ��� ������� ������ ����� �� ��� ���� ��� ������’� ��������� ���������� �������� ���� ������ ������ ���������� �������� ������� �� ����������� ������ ���� ���� ���� ����.

(�) ��� ������� ������ ��������, ������� ��������� �������������, ���� ����� ��� ���� ��� �������� ������������ ��� ��� ��������� �� ��������.

�. ������� ��� ��������� �� ��� ������� ���� ������ ������ ��� ��� ����� ��������� �� ���������. �� �� ��������� ���� ���� ��� ���������� ��� ��� �������� �� �� ���� ������. ���� ����� �� �����������, ��� �� ���� ���� ��� ��������� ���� �� �����. ��� ���� �� ��� ��������� ����� ��� � ������.

����� ��� �� ��� ��������� ����������, �� �����, ���� ����� �� ������� ��� ��������� �� ��� ������’� ���������

(�) ����� ������ �� ������ �� ��� ���� ���� ���������� ������� ��� �����.

(�) ����� ��� ���������� �� ���� �� ���� ��� �������.(�) ���� �������� ������ ���� ����� ����� �� �����

��������� ����������.(�) ����� ��� ���������� �� ���� ������� ������ �����

�����.(�) ����� ������ ��� �� ���� �������� ������ � ������

�����.

GO ON TO THE NEXT PAGE.

73

Page 14: Official LSAT PrepTestdocshare02.docshare.tips/files/27057/270577487.pdfLSAT® Writing Sample Topic Directions: The scenario presented below describes two choices, either one of which

2 -14- 2 2 2 �. ����� � ��� �������� ���� ������� �� ��� ����������, �����

�, �� ��������� ������������� �������� �������������, ����� �� �������. ���� ����������� ���, �������, ��� �������. ����� ����� ���, ����� � ������ ��� �������� �� � ������� �� ����� �� ��� ���������� ���� ��� �������� ��������� ��������� �������� �������� ����.

����� ��� �� ��� ��������� �������� ������ ��������� ���� ������� �� ��� ������ ��������� �� ��� �������� ������

(�) ��� ��������� ������� ��� ��������� �� ��������� �������� ����, ��� ��� ����������� ��� ��� �������. ����� ��� ���������’� ������� ��� ������� ������� ����, ���� ��� ��� �������, ������� ��� ���� �� �������� ��� ������.

(�) ��� ��������� ������� ��� ��������� �� ��������� �������� ����, ��� ����� ����������� ���� ��� ��������� �� �� ������������, ������� ��� ��������� ��� ������� �� ������� �������.

(�) ��� ��������� ������� ��� ��������� �� ��������� �������� ����, ���, �� ��� ����, ������ ���� �������� ���� ���������� �������. ������������, ������� ��� ��������� �������� ������� �� ������� ���� ���� ������� �� ����� �� ��� ���������, ��� ���������’� ����������� ��� ��� �������.

(�) ��� ��������� ������� ��� ��������� �� ��������� �������� ����, ��� ����� ����������� ��� ��� �������. ���� ��� ������� �� ������� �� ���� �� �����������, ������� ���� ���� ���� ���� ��� ����� ������ ��� ��������.

(�) ��� ��������� ������� ��� ��������� �� ������ ���� ��� �������� ���� ���� ����� ����� ��� ���������’� �����. ���� ���������� �� ��� �������, �������, ������� �� ������� ��� ���������’� ����������� ������� �� ���������� ��� ��������� ��� ��������� ��� ��� ����� ���������� ����� �����.

�. ���������� ������ ������� ����� �� ��� ��������� ���� ���� ���� ����-������ ������ ������� �� ��������� ����� ������ ���� ������ ���� ������. ��� ��� ��� ���������’� ������� ��� ��� ��� ������� �� ������� � ����� �����, �� ����� ��������� ��� ������� ���� � ������ ����� ���� ������� ��������� �������� �� ������� ��� ��� ���� ������. ���� ������� ����� ���� ���� ��� ������� ���� �� �� ������’� ����� ������ ���� �� ���� �������� ����� �� ��� � ���� ��� ����� �� ��������.

��� �������� �������� �������� ����� ��� �� ��� ����������

(�) ��� ��������� ��� ��� ���� ���� �� ��������� ���� ������� ���� �� ��� ����� �������.

(�) ��� ��������� ���� � ���� �� ������ ��������� �������� �� ������� ��� ��� ���� �������.

(�) ��� ��������� ����� ������� ���� ������ ������� ���� ������� ����� ���� �� ����� ���� �������.

(�) ��� ��������� ��������� ���� ��������� ����� ������� �������� ����� ������.

(�) ��� ��������� ��� ������ �� ����� ���� ����� �� ��������� ����.

�. ���������� �������� ����� ����������� ���������� �� ��� ������ �� ������ �������, ��� ���������� �� ��� ������ �� �����-���� ������� �� ����� ��������� ���� ���� ������� ��������� ������������ �� ������� �����������. ���� �������� ����������� ������� ��� ��������� ������� ���������’ ������ ���� ����������� ��� �� ����������� ������ ������ �� ��� �������� �� �����.

����� ��� �� ��� ���������, �� ����, ���� ���������� ��� ���������’� ����������

(�) ������ ��� ����� ��� �������� �� ���� ������ ������� ���� ��� �� ������ ������� �� ������� �����������.

(�) ��������� ����� ���� �� ���� �������� ������������ �� ������� ����������� ���� �� ����� �����.

(�) ������������ �� ������� ����������� ��� ������� ���� �� ��������� ����� � �������� �������� ������ ������� ��� �� ������� ���������� ��� ����������.

(�) ���� ������ ��� ����� ������� ������ ����������� �������� �� ����� ���������� ����� �����.

(�) ������ ��� ��� ������� ���������� �� ������� ����������� ���� �� �� ���������� �� ����� ����� �� ����������� �� ����.

GO ON TO THE NEXT PAGE.

73

Page 15: Official LSAT PrepTestdocshare02.docshare.tips/files/27057/270577487.pdfLSAT® Writing Sample Topic Directions: The scenario presented below describes two choices, either one of which

2 2 2 -15- 2 ��. ������ ������� ������’� ����� ��� ��� ��������� ����������

������. ��� ����� �� ������’� ����� ������� ���� ����������� ������� ��� ������� ���� ���������, �� ���� �� ��� ������ ������� ��� �����, ���������� ���� �� ���������, �� ����, �� ������� ��� �� ��� ����������’ �������������. ���, ��� � ���� �� ������� �� � �����, ��������� ���� ���� ���� ������� �� �� ����� ���� �� ��� ����������.

��� ���������� �� ��� �����’� �������� ��� �� �������� ����� �� ����� ��� �� ��� ��������� �� ��������

(�) �� �������� ���� ������ ������� ������� � ���������’� ����������� ���� ��� ���� ��� �������� �� ���� ���������.

(�) � ���������’� ����������� �� ���������� ��������� �� ��� ������� ��� ������� �� ���� ���������.

(�) ��� ������ �� ����� � ���� �������� �� � ����� �� �������� ������������ �� ��� ������ �� ����� ��� ����’� ��������� ���� ����� ��� ����������.

(�) �� ��� ������������� �� � ����’� ���������� ��� ��� ������� ����������� �� ��� ������ ������� ��� �����, ���� ����� ������������� ��� ��� ������� ����������� �� ��� ����’� ��������.

(�) ��� ����� ����, ������ ������’� �����, ���� ���������� ���� ���� ��������� ��������� ������� �� ������.

��. ��������� ����������� ��� ����� �������� ���� ��� ���� ������ � �������� �������’� ���� �� ������� ����-���� ������������. ������ ����� ����� �� �� ������� �� ��������� ����� ������� ���� ������’� ���� ���� ��� ������� ����� �� ��������� ��� ����’� �������� ��������� ���� �������� ���������. ��� ���� �������� ������ � ��� ���� �� ��� �������’� ������ �� ���� ��� �������� ���� �� ������ ��� ���� ����� ��� ���� ��� �� �������� ����������. �� ��� ����, ���������� �� ���� ��������� �� ��������� �� ��� �����������-������� ���������.

����� ��� �� ��� ��������� ���� ���������� ��������� ��� ���� ���������� �� ��� ��������� ����������’� ���������

(�) ���� ������’� ���� ���� ��� ������� ����� �� ��������� ��� ����’� �������� ��������� ���� �������� ��������� �� ���������.

(�) ��� �����’� �������� �� ������ ��� ���� �� ������������ ������ ��� �� ���������� �����������.

(�) �� �� ��� ����������� ���� ��� ������� ������ ���� ��� ������ ����������� �� ������� ��� �������� �� ������ ��������� ��� �������� ����������.

(�) ��� ����’� �����������-������� ��������� �������� ���������� �� ��� ���������� �� ���� ���������.

(�) ��� �������� �������’� ������ �� ������� ��� �������� �� �������� ������ �� ��� ���� ���������� ������ ��� ��������� ���� �� ������������.

��. ��� ����������� ������ ��� ���� �������� ��� �������� ����������’� ������ �� ��� ������, ��� ��� ����������� ��� ��� ������� ��� ����� ������� �� ��� ���������� ����� ��������� ��� ������ ������ ������ �� ��������.

��� ��������’� ���������� ��� �� �������� �������� �� ����� ��� �� ��� ��������� �� ��������

(�) �� ����� ���� ���� ����������� ��� ��� ����������� �� ������� ��� ����������’� ������ �� ��� ������ ���� �� ���� ����� ������� �� ��� ���������� ��� ����� ����� ����� �������.

(�) ��� �� ��� ������� �� ��� ���������� ��� ������ ��� ������ ����� �� ��� �������.

(�) ��� ����������� ����� ��� ���� ���� ��������� �� ��������� ��� ����������’� ������ �� ��� ������� �� ��� ���������� ��� ������� ������������ ����� ��� ������’� �������.

(�) ��� ����������� ����� ���� ���� ��������� �� ��������� ��� ������ ���� �� ���� �� ��� ����������’� ������� ����� ���� ������ �� ��� ����� �������� ��� ���� ���� ���������.

(�) ���� ������� �� ��� ���������� ����� ���� ��������� ���� ��� ������ ��� �� �������� �� ��� ������.

��. ��������� � ������ �� ������ ����������� ����� ����� �� ��� �������� ����� ����, ����� �� ��� ������ ������ �� ������ ��� ���,��� ������. ��� ������ ����� �� ����������� ��������� �� ��� ����� ���� �� ��� ���� �� �����, ��� ��� ���������� �� ��� ���������� �� ������ ��� ��������� �������������, ��� ������ ����������� �� ������� ��� ����� �� �� �������� ����. ���� ������������ ���� ������� ���� ������ �� ������ ������ ���� �� ������ �����.

� ���� �� ��� ��������’� �������� �� ���� ��

(�) ������ ������� ������������� ���� ������� ��� �������� ����� ���� ��� ���������, ��� ������ �� ������ �������� �� ���� ������ ���������� ��� ���������

(�) ������� ��� ����������� ���� ��� ����� ���� ����� ���� ������������� ��������� �� �� ��� ��� ���� ��� ��� ������� ���� �� ������������

(�) ��������� ��� ����������� ���� ��� ���������� ��� ��������� ������������� ���� ��� ���� �� �����

(�) ��������, ������� ��������� �������, ���� ����������� �������� ��� �������� ����� ����� �� ���� ��������� ���� ������������

(�) ����� ��� ������� ���� ��� ������ �� ��������� ���� �� ������������ �� ��� ������ �� ������ ���������

GO ON TO THE NEXT PAGE.

73

Page 16: Official LSAT PrepTestdocshare02.docshare.tips/files/27057/270577487.pdfLSAT® Writing Sample Topic Directions: The scenario presented below describes two choices, either one of which

2 -16- 2 2 2 ��. ����� �����-����������� �������� ��� ��� ����������

���� ����� ����, ����� ���� �������� ������ �� ������������� �������� ���� ��������, ��������� ������������. ����� ����������� �� ���� ���� ��� ������ ��� ��������� ��� ��� ��� ����� ���������� �� ����� ����������� �� ���� � ���������� ���������� �� ���������� ������ ������ ���������� ���� �� ���’� ������ ��� �� ������ �� ����� �����������.

������� �� �� ������ �� ��� �� ������� ������� ��� ����� ����������� ������ �� �������� ��� ���� ���� �������� ���� ���� �� ������������� ��������. �� ���������� ���� ��������� ������������ �� ��� ���� �� �������, ���� �� ������ �� ������ ��������.

������ �������� �� ���� ��

(�) ������� ���� ����� �� �� �������� ���� ��� ������� ���� ����������� ���� �� ���� �� ��������

(�) ���������� ���� ���� ��� ���������� �������� ���� �������� ������ ������� ��� �������� ���� ��������

(�) ������������� ���� ��� ������ �� �������� ���� ���� ����� ���������� ���� �����

(�) ��������� �������� ���� ��� ���������� ������� ���� ���� ������� ��� �� �������� ���� �� ����� �������

(�) �������� ���� � ���� ���� ���� �������� ����� �� ������� ������� ��� �������� ���� ��������

��. ��������� ����������� ���� ���� � ��������� ���� ���’� ������ ����������� �������� ����� �� ��������� ���� ��������� ��� ����������� �� ��������� ��� ��������. ��� ��� ������� �� ����������� ��� ���� ��� ������ �� � ��������� ����������. ���� ������’� ����� ������� ���� ���� ����� ��� ���� ��� ���’� ��� �������� ��� ������� ���������� ������. ����, ��� ����������� �������� ��� ��� ���������.

��� ��������� ����������’� ��������� �� ���� ���������� �� ��������� �� ��� ������� ����

(�) �� ����� ��� ������� ���� ���’� ����� ����� ��� ���� ���� ����� ����� �� ��� ������� �� ��� ����������’� ����������� ��������

(�) �� ����� �� �������� ���� �������� ������� ��������� �� ��� ����������� �������� ��� ���� ������ ���’� ��� ����� �������

(�) �� ����� ��� ������� ���� �� ���’� ��������, ��� �������� ������ ������� ����������� ��������

(�) �� ����� ��� ������� ���� ��� ������ ����� �� ����������� �������� ��� ��� �� ��������� �����������

(�) �� �������� � ��������� ��������� ��� ���������� ������� ����� ���� � ��������� ���� ���� ������ ��������� �����

��. ��� ��� ����� �� ������������, ��� �������� �����, �� ������� ��� ���������� �����������, ���� ����� ������ ��� �������. ��� ���������� ��� ������ �� ����� ���������, ��� ��������� ��� ��� ������ �� ��������. �� ������������, ��������� ��� ��� ���� ���������� �����������, ��� ���� ������, ����� ������� ��� ������ ��� ���� �������. ����, �� ����� �� ������ �� ��� ��� ����� �� ������������ ���� ������� �� ��� ���� �������� ������ ���� ��� ���� ���������.

��� �������� �������� ��

(�) �������� � ���������� ����� ��� ��� ��������� ������ �� ���� �� ��� ����� �� ��� ����� �� ����������� ���� ������������� ��������� �� ����� ������

(�) ������ � ����������� ������� ��� ��������� ����� �� ������� �� ����� �� �������� ���� ��� ��� ���� �������� ����� ���� ��� �����

(�) �������� �� ������������ ������� ��� ��������� �� � ����� ��� ���������� ���� ���������� �� ��� �������� ��� ������� �� ������� �� ��� �����

(�) ������� ���� ������� ��� ��������� ������ ��� ������������, ��� ��������������� �� ��� ����� ��� ��� �������� �� ���������� � ���������� ����� ��� �����

(�) ���������� ���� �� ������ �� ������������� �� ���������� �� �������� ���� � ������������� ������ �� �� ��������� ���� �� �������������

GO ON TO THE NEXT PAGE.

73

Page 17: Official LSAT PrepTestdocshare02.docshare.tips/files/27057/270577487.pdfLSAT® Writing Sample Topic Directions: The scenario presented below describes two choices, either one of which

2 2 2 -17- 2 ��. �� ������ �� ��� ��� ���� ��������� �� ��� ���������� ���

�������, ���� �������� ��. ����, �� ��� ���� ��������� ��������� �� ������� ��� ������ �� ��� �������, ���� �������� ���� ��.

��� ��������� �� ����� ��� �� ��� ��������� �� ���� ������� �� ��� ��������� �� ��� �������� ������

(�) �� ��� ���������� �������� ���� ��� �� �� ��� ������ ������, ���� �� ���� �� �� ��������. �� �� ��� ���� ��� �������� ��� �������� ���� ���, ���� ��� �������� ���� ���� ���� ������� �� ��� ������ ������.

(�) �� ��� ������ ������ �� ��� ���������� �������� �� ��� ������, ���� �� �� ������. �� �� ��� �������� ���� �� ��� ������ ������ ��� �� ���� ��� �� �� ������, ���� �� ���� �� �� ������.

(�) �� ������� �� ��� ������� ��� ����������� ��������, ���� ������� ���� ��. �� �� ��� �������� ���� �� ��� ������ ������ ��� �� ���� ��� �� �� �������, ���� �� ���� �� �� �������.

(�) �� ������� ��� ��� ������ ��� ������ ��� �� ��� ����������� ��������, ���� ������� ��� ������. �� �� ��� �������� ���� �� ��� ������ ������ ��� �� ���� ��� �� �� ������, ���� �� ���� ��� �� �� ������� ������.

(�) �� ����� �� ��� ��� ������ ������ �� ��� �������� ��������, ���� �������� ��. �� �� �������� ���� ��� ��� ��� �������� ��� ����� ���� ��� ��� �� ������, ���� �� ���� ��� �� ������� �� ��� ������ ������.

��. ������� �� ��� ��������� ������ ���� ������� ���������-������� �������� ��������� ���� ��������� ���� ����������� (���� ��, ���-����������� ���� �� ������� �� ���������� �����) ���� ���� ���������-������� ���������, ���������-������� �������� ��� � ������� ������� �� �������� ���� ��� ���������-������� ��������. �������, ���� ���������� �� �����. ���������-������� �������� �������� ��� ��� ���� � ������� ������� �� ��������, ��� ��� ������ �� ����� � �������� �� ����������� �� ���������� �� ��� �������’� �������.

��� �������� �� ������ �� ���� ��

(�) ������� � �������� ������ ������� ��� ��������� �� ��� �������� ��� ����� ������������� �����

(�) ��������� ������ �� ��� ��������� �������� �� ��� ���� “�������”

(�) ����� � ��������� ��������� ��� �� ��������’� ����� ���������� �� �� � ���������� ��������� ��� �� ��������’� ����� ����������

(�) ����� ��� ���������� �� ��� ������ ���� ���������� ���� �����

(�) ������� � �������� �� ��� ������� ���� �� ���������� �������� ��� ���� ���� ��� ��

��. � ������ ����� ������ ��������� � �������� ����� ���� ���� �� ���� ����� ��� ����� �� � ����� ��� ���� �� �������. ���� �������������� ������� ���� ��� ������ ��� � ������—��� ���� �� � �������’� ����—��� �� �� ��� ����� ��� ���� �������. ������� �� ��� ���� ��� ��� ���� ���� �� ���� ����� ���������� ��������, ��� ������ ��� �������� ��� ���� �� � �������� �����, � �������� ������ ������ ������ � ����� �������� �� �������� ��� ��� ��� ����� �� �����.

����� ��� �� ��� ���������, �� ����, ����� ���� ������ ��� ���������

(�) ��� ���� �� ����� ��� ������ ��� ����� ��� ��� ������� ��� ����� ������� ���� ����� ���� ���� �������.

(�) �������� ������� ���� �������� ������ ���� ��� ���������� �� ��� ���� �� ����� ��� �������.

(�) ��� ������ ��� ������ ���� �� �������� ���� ��� ���� �� ����� ��� �������.

(�) ��� ���� �� ����� ��� ������ ��� ����� ��� ���� �� � ����������� ��������� ������.

(�) � �������� ����� ���� � ����� ���� �� ������� �� ��������� � �������’� ����.

��. ��� ������ �� �������� ����������� ��� ��� ������� �� � ������� ������ �� ������� ��� �������� �� ������������ ������� � “�����-������” ����, ���� �� �������, �� � ����� �� ���������� ��� ����. �� � ������, ��� ���� ��������� �� � ������� ���� ����� �� ��� ������ �� ����. �� �� ������������� ������� ��� ���� ���������, ������� ���� ���� �� ������� ��� ��� �� �������� �����������.

��� �������� ������ �� ��� ���������� ����

(�) ����, �� ��� ���, ������� �� ��� ������ ��� ������� ��� ��� �� �������� ����������� ���� ������������ ���� �������

(�) �������� �������� ����������� �� �����-������ �����, ���� �� �������, ��� �� �������� ������ �� ����� ������

(�) ��� ���� ��������� ������ ����������� ��� ���� ������� �� � ���� ����� �� ���� ���������

(�) �������� ����������� ���������� ���� � ����������� ������ �� ��� ���� ��������� �� ���� ������

(�) ����, �� ��� ���, ������� �� ��� ������ ���� ��� ���� �����-������ ����� ������ ���� ������� ��� ��� �� �������� �����������

GO ON TO THE NEXT PAGE.

73

Page 18: Official LSAT PrepTestdocshare02.docshare.tips/files/27057/270577487.pdfLSAT® Writing Sample Topic Directions: The scenario presented below describes two choices, either one of which

2 -18- 2 2 2 ��. ���� �� ��� �������� ��� ���� ������� ��� �� ���

���������� ���� �������� �������� ����� ����� �������. �������, ���� ������� ��� �������� � ����� ����� ���� � ����� ������ �� ����� ��� ����� �������.

����� ��� �� ��� ��������� ���������� ����� ��� �������� ��� ���� ������� ��� �� ��� ���������� ���� �������� ��� �� �������� �������� ���� ��� ����������� ������

(�) �� ����� ���� �� ��� �������� ��� �������� � ����� �� � ����� �� ������ �������� ����� ����� �������.

(�) ����, �� ��� ���, �� ��� �������� ��� ������ �� ����� ��� ����� ������� �������� � ����� ����� ���� � �����.

(�) ���� �� ��� �������� �������� � ����� ������ ���� � �����.

(�) �� ����� ��� ������� ��� �������� � ����� �� � ����� �� ������ ������ ��� �� ���� ����� ��������.

(�) ���� ���� ���� �� ��� �������� �������� � ����� �� � ����� �� ������.

��. ������� ��� ������ ������ �� ���� ���������� ��� ������ �����������, ������� ������ ���� ���������� �� ����. ����� ����� ����������, ���� ����������� ������� ����� �� �������� ����� ���� ��� �������� ����������� ���� �� ��� ����������, ��� ���������� ���� ����� �� ��� ���� ��� ��� ����� ���������� ������� �����. ����� ��� ��� ����������� ��� ������ �����������, ���� ����������� ����������� ���� ���� ��� ������� ����������� �� ��� �������� �������.

����� �� ��� ���������, �� ����, ���� �������� �������� ��� �����������’ �����������

(�) ������� �� ��� ��� ����������� �� �������� ��� ��������� ���� ��� ������ ������.

(�) ���� ��� ������ ������ �� ���� ���������� ���� ��������, ���� ��������� ��� �������� ����������� ���� ��� ��� ����������.

(�) ���� ����� �� ������ ���� ������� ����� ����� ���� �� ��� ���� �� ������� ����� ��� ���� �� ������ �����.

(�) ��� �� ��� ����������� �� �������� �� ��������� ��������� ����������� �� ��� �������� ���������� ���������� �� ����.

(�) ��� ����� ������ �� ������� ������ �� ��� ���� �� ��� �� ����� �� ��� ������ �� ������ ������ ��� ���� ���� ����� ���.

��. � ���������� ������� ��� ������������� �������� ��� �������� ������ �� ��� �������������� ��� ������� �� ������ �� ������ ���������� ���� ���� ��� ��� ���� ������� �����. ��� ����� ��� ����� �� ������ �� �� �������� �� ���� �����. ��������, �������������� ���� ������ ��������� �� ���� ���� ���� ����� �� ��� �������� ��������, ���� ����� ������ ���� ������ �� ��� ���� ��������, ��� ��� ����������� ���� ��� ���� ����� �������.

��� ���������� �����, �� ����, ���� �������� ������� ����� ��� �� ��� ��������� �������

(�) ���� � ���������� ������� ��� �� ����� ��� �������� �� � ������ ��������, ������� ���������� �� ���� �������� ���� ��� ���������� �� ��� �������’� �������� ������.

(�) �� ���������� ���������, ����� �� ������� ����������� ������ ������ ���������� ���� ������ ����������� ����������.

(�) � ���������� ������� ��� �������� ��� ��������� ��� �������� ������ �� ��������� ����� ��������� ��� ���������� �� ������ ���������� ���� ���� ��� ��� ���� ������� �����.

(�) � ���������� ������� ���� ��� ���������� �������� ������ ������ ��� �������������� ������ �� ������ ����������.

(�) ����������� �� � ������ �������� �� � ���������� ������� ����� ������ ���� �� ���� �� ��� ������� ��� �� ����� ��� ����� �������� �� ���� ��������.

GO ON TO THE NEXT PAGE.

73

Page 19: Official LSAT PrepTestdocshare02.docshare.tips/files/27057/270577487.pdfLSAT® Writing Sample Topic Directions: The scenario presented below describes two choices, either one of which

2 2 2 -19- 2 ��. � ������ �� � ����’� ������������ ����� ���� ������ ��� ��

���� ���� ������������ ���� ��� ����� ������� ����. � ����� �������� �� ���� ��������� � ������ ���������� ��� ����� ����� ��� ������ ���������. ���, ���� ������ ��� ������ ����������� ���� ���� ���� ��� �������� ������� ���� ������ �������� �� �������� �� ������� ����� ��������, ���� �� ���� ������� ��� ���� �� ������� ���� ��� �������� ��������� ��� ��������� �� ���� � ������� ���� ���� ����� ����� ��� ������ ���������.

����� ��� �� ��� ���������, �� ����, ���� ����� �� ������� ��� �������� �������� �� ��� ������������’ �����, �� �������� �� ��� �������

(�) ������ ��� �� ��� ������ ��������� ���� �����, ��� ����������� ���� �������� ���� ��� ������ ��� ��������� �� � ����� ���� ��������� ��������� ��� �������� ������� ����.

(�) ���� �� ��� ������ ��� ���� �� ��� �������� �� ��� �������� ������� ���� �� ��� ���� �� �� �� ���� ����.

(�) ��� ����’� ������������ �������� ���� ������� ���� �������� ������� ���� ��� ������������ �� � ��� ������� ���� ���� ���� ����������� �� ��� �������� ������� ����.

(�) � ����-���������� ���� �� ����� ���������� �� ��� ���� ���������� ���� ����� ������� ��� �������� ������� ���� ���� � ������ ���������� ��� ����� � ��� ������� ���� ������.

(�) ���� ������� ������� ��� ��������� ��� ��������� �� ��� ���� �������� �� ��� ���� ����� ����� �� ���� �������� ����� �� � ��� ������� ���� ���� �����.

��. �������� ������ ���������� �� �������� �����, � ���� �� ���� ��� ���� ���� ����� � ������ � ������� �� ����� �� ��� �����. ������� ��� ����, � ���� �� ������ �� ����� �� �������� ��������, ��� ������� �� �������� ��������, � ���� �� ������ �� ������� ��� ���������. �����, ����� ��� ��������� ����� ���� �� ���� ������ ���� ��� ��������� ���� �������, �������.

����� ��� �� ��� ��������� ���� ��������� ��������� ��� �������’� ���������

(�) � ���� ���� �� ������� �� ���������� ��������(�) � ���� �� ������ �� �������� �� �������� �����(�) �� � �� ��� ���� ��� ����, �� �������� ����� ����

������(�) �� � �� ��� ���� ��� ����, � ���� ������� ���

��������� ������� �� �������� ��������(�) �� � �� ��� ���� ��� ����, � ���� �� ������ ��

�������� �� �������� �����

S T O PIF YOU FINISH BEFORE TIME IS CALLED, YOU MAY CHECK YOUR WORK ON THIS SECTION ONLY.

DO NOT WORK ON ANY OTHER SECTION IN THE TEST.

73

Page 20: Official LSAT PrepTestdocshare02.docshare.tips/files/27057/270577487.pdfLSAT® Writing Sample Topic Directions: The scenario presented below describes two choices, either one of which

3 -20- 3 3 3 3SECTION III

Time—35 minutes23 Questions

Directions: Each group of questions in this section is based on a set of conditions. In answering some of the questions, it may beuseful to draw a rough diagram. Choose the response that most accurately and completely answers each question and blackenthe corresponding space on your answer sheet.

��������� �–�

� ������ �������� �� �������� ��� �������� �� � �� ���������� �� � �������� �� ������� ���� ������������ ������—�����������, ��������, ���������, ���������, ��� ��������. �� ������ ��� ������� ������� �����, ��� �������� �� ������ ���� ������� ��� ��������� ������������

�������� ���� �� ������� ���� ���������.��������� ���� ������ �� ������� ���� ���� ����������� ��� �������� �� ����� ���� ���� ����������� ��� ��������.

�������� ���� ������ �� ������� ���� ���� ����������� ��� ��������� �� ����� ���� ���� ����������� ��� ���������.

�. ��� ���� ������ ����� ������ �� ����� ��� �� ��� ��������� ��������� �� ��� ��, �� ����� ���� ����� �� �����

(�) �����������, ���������, ��������, ��������, ���������

(�) ��������, �����������, ���������, ���������, ��������

(�) ���������, ��������, ��������, ���������, �����������

(�) ���������, ��������, ��������, �����������, ���������

(�) ��������, ��������, ���������, ���������, �����������

GO ON TO THE NEXT PAGE.

73

Page 21: Official LSAT PrepTestdocshare02.docshare.tips/files/27057/270577487.pdfLSAT® Writing Sample Topic Directions: The scenario presented below describes two choices, either one of which

3 3 3 3 -21- 3 �. �� �������� �� ��� ������ ����� �� ��� ��, ���� �����

��� �� ��� ��������� ���� �� �����

(�) ����������� �� ������� �� ��� �� ���� ��������.(�) �������� �� ������� �� ��� �� ���� ���������.(�) ��������� �� ������� �� ��� �� ���� �����������.(�) ��������� �� ������� �� ��� �� ���� ��������.(�) �������� �� ������� �� ��� �� ���� ���������.

�. �� ����������� �� ��� ����� ����� �� ��� ��, ���� ����� ��� �� ��� ��������� ����� �� �����

(�) ��������� �� ��� ������ ����� �� ��� ��.(�) ��������� �� ��� ����� ����� �� ��� ��.(�) �������� �� ��� ����� ����� �� ��� ��.(�) �������� �� ��� ������ ����� �� ��� ��.(�) ��������� �� ��� ���� ����� �� ��� ��.

�. �� ��������� �� ��� ������ ����� �� ��� ��, ���� ����� ��� �� ��� ��������� ����� �� �����

(�) �������� �� ��� ����� ����� �� ��� ��.(�) ����������� �� ��� ����� ����� �� ��� ��.(�) ��������� �� ��� ����� ����� �� ��� ��.(�) �������� �� ��� ������ ����� �� ��� ��.(�) ����������� �� ��� ���� ����� �� ��� ��.

�. ��� ����� ��� ������ ������ �� ��� ��, ������ �� �����, ����� ��

(�) ����������� ��� ���������(�) ����������� ��� ��������(�) �������� ��� ���������(�) ��������� ��� ��������(�) �������� ��� ��������

�. �� ��������� �� ��� ������ ����� �� ��� ��, ���� ����� ��� �� ��� ��������� ����� �� �����

(�) �������� �� ��� ����� ����� �� ��� ��.(�) �������� �� ��� ����� ����� �� ��� ��.(�) ��������� �� ��� ����� ����� �� ��� ��.(�) ����������� �� ��� ������ ����� �� ��� ��.(�) ����������� �� ��� ���� ����� �� ��� ��.

�. �� �������� �� ��� ����� ����� �� ��� ��, ���� ����� ��� �� ��� ��������� ������ �� �����

(�) ��������� �� ��� ����� ����� �� ��� ��.(�) ��������� �� ��� ����� ����� �� ��� ��.(�) �������� �� ��� ������ ����� �� ��� ��.(�) ��������� �� ��� ������ ����� �� ��� ��.(�) ��������� �� ��� ���� ����� �� ��� ��.

GO ON TO THE NEXT PAGE.

73

Page 22: Official LSAT PrepTestdocshare02.docshare.tips/files/27057/270577487.pdfLSAT® Writing Sample Topic Directions: The scenario presented below describes two choices, either one of which

3 -22- 3 3 3 3��������� �–��

�� � �������� ��������� ����� ���� �� ������� ���� ��������� ����, ������, ����, �������, ��� ���������. ���� ������� ���� ���� ������� ��� ������, �� ������ ��� ���� ���� �� ��� ���� ����. �� ���� ����, ����� ���� �� ������� ��� ������ �� � �.�. ��� ��� ������ �� � �.�. �� ��� �� ��� �����, ��� �� �� ����������, ����� ���� ���� �� � ������ �� � �.�. ��� �������� �� �������� �� ����������� �� ��� ����������

������’� ������ ���� �� ������� ���� ����’�, ��� �� ��� ���� ����.

������� ����’� ������ ��� �������’� ������ ��� �� ������� ���� ���������’�.

�� ����’� ������ �� �� ��� ���� ����, ���� ����’� ��� ���������’� �������� ���� ���� �� �� ��� ���� ����.

�. ����� ��� �� ��� ��������� ����� �� ��� �������� ����� �� ���� ����, ������ �� ��� ����� �� ����� ���� ������

(�) ���� ����� ������’�, ����’����� ����� ���������’�, ����’�, �������’�

(�) ���� ����� ������’�, �������’�, ����’����� ����� ����’�, ���������’�

(�) ���� ����� ����’�, ������’�, ����’����� ����� ���������’�, �������’�

(�) ���� ����� �������’�, ����’�, ������’����� ����� ���������’�, ����’�

(�) ���� ����� ���������’�, ������’����� ����� ����’�, �������’�, ����’�

GO ON TO THE NEXT PAGE.

73

Page 23: Official LSAT PrepTestdocshare02.docshare.tips/files/27057/270577487.pdfLSAT® Writing Sample Topic Directions: The scenario presented below describes two choices, either one of which

3 3 3 3 -23- 3�. ����� ��� �� ��� ��������� ����� �� �������� ������

�� ����� �� ��� ���� �����

(�) ����’� ��� �������’�(�) ����’� ��� ���������’�(�) ������’� ��� ����’�(�) ����’� ��� �������’�(�) �������’� ��� ���������’�

��. �� ����’� ������ �� �� � �.�., ����� ��� �� ��� ��������� ������ �� �����

(�) ����’� ������ �� �� ��� ���� ���� �� �������’�.(�) ������’� ������ �� �� ��� ���� ���� �� ����’�.(�) ����’� ������ �� �� ��� ���� ���� �� �������’�.(�) ����’� ������ �� �� ��� ���� ���� ��

���������’�.(�) �������’� ������ �� �� ��� ���� ���� ��

���������’�.

��. ����� ��� �� ��� ��������� ����� �� � �������� ��� �������� ���� �� ��� �������� ����� �� ��� ���� ����, �� ��� ����� �� ����� ���� ������

(�) ����’�, ������’�(�) ������’�, �������’�(�) ������’�, �������’�, ����’�(�) �������’�, ���������’�, ����’�(�) ���������’�, ������’�, ����’�

��. �� �������’� ������ �� �� � �.�., ����� ��� �� ��� ��������� ����� �� �����

(�) ����’� ������ �� �� � �.�. �� ��� ���� ����.(�) ����’� ������ �� �� � �.�. �� ��� ���� ����.(�) ������’� ������ �� �� � �.�. �� ��� ���� ����.(�) ����’� ������ �� �� � �.�. �� ��� ���� ����.(�) ����’� ������ �� �� � �.�. �� ��� ���� ����.

��. ����� ��� �� ��� ���������, �� ����������� ��� ��� ���������� ���� ������� ����’� ������ ��� �������’� ������ ��� �� ������� ���� ���������’�, ����� ���� ��� ���� ������ �� ����������� ��� �������� �� �������� ���� ������ �� ����� ��� ������

(�) ����’� ������ ���� �� �� � �.�.(�) ������’� ������ ������ �� ������� ����

���������’�.(�) ������ ����’� ������ �� �������’� ������ ����

�� ����� ���������’�.(�) ������ ����’� ������ �� �������’� ������ �� ����

���� �� �� � �.�.(�) ���������’� ������ ���� �� �� � �.�.

GO ON TO THE NEXT PAGE.

73

Page 24: Official LSAT PrepTestdocshare02.docshare.tips/files/27057/270577487.pdfLSAT® Writing Sample Topic Directions: The scenario presented below describes two choices, either one of which

3 -24- 3 3 3 3��������� ��–��

������ ��� ����������� �������, ����� ��������—��� ������, ��� ����������, ��� ��� ��������—����� ��� ���� ��������� ���� ����������� ��� ������ �� ����� �������—��� �����, ��� �������, ��� ���, ��� ����, ��� ��� ������. ���� ������ ����� �� ����� ��� �� ��� ��������� ��� ���� �������� ��� ����� �� ������� ��� �� ��� ��������. ��� ���������� �������� ����������� ��� ��������� ����� ��� ��������� �� ��� ����������

��� ���������� ����� ���� �� ��� ��������� ���� ��� �������� �����.

������� ��� ��� ��� ��� ���� �������� �� ��� ����� �� ��� �����.

������ ��� ������ ����� ��� ������ �� ��� �������� ����� ��� ���, �� ����.

��. ����� ��� �� ��� ��������� ����� �� �� �������� �������� �� ���� ������ �� ��� �������� �� ��������� �� ������

(�) ������� ��� �������, ��� ����������������� ��� ���, ��� ������������� ��� �����

(�) ������� ��� �������, ��� ��������������� ��� ���, ��� ��������������� ��� �����

(�) ������� ��� �����, ��� ��������������� ��� �������, ��� ��������������� ��� ���

(�) ������� ��� �����, ��� ������������������ ��� ������������� ��� ���, ��� ������

(�) ������� ��� ����������������� ��� ���, ��� ������������� ��� �����, ��� �������

GO ON TO THE NEXT PAGE.

73

Page 25: Official LSAT PrepTestdocshare02.docshare.tips/files/27057/270577487.pdfLSAT® Writing Sample Topic Directions: The scenario presented below describes two choices, either one of which

3 3 3 3 -25- 3 ��. ����� ��� �� ��� ��������� �� � ���� �� ��������� ����

������ ���� ���� ���� ����� �� ��� �������

(�) ��� �����, ��� �������(�) ��� �������, ��� ����(�) ��� �������, ��� ������(�) ��� ���, ��� ����(�) ��� ���, ��� ������

��. �� ��� �������� ����� ��� ����, ����� ��� �� ��� ��������� ���� �� �����

(�) ��� ������ ����� ��� �����.(�) ��� ������ ����� ��� ���.(�) ��� ���������� ����� ��� �����.(�) ��� ���������� ����� ��� �������.(�) ��� ���������� ����� ��� ���.

��. �� ��� �� ��� �������� ����� ���� ��� ������� ��� ��� ���, ����� ��� �� ��� ��������� ����� �� �����

(�) ��� ������ ����� ��� �������.(�) ��� ������ ����� ��� ����.(�) ��� ���������� ����� ��� �����.(�) ��� ���������� ����� ��� ������.(�) ��� �������� ����� ��� ���.

��. �� ��� ������ ����� ������� ��� �� ��� ���������, ����� ��� �� ��� ��������� �� � �������� ��� �������� ���� �� ��� ��������� ��� ��� �� ����� ����� �� ��� �������� ���� ��� ������ ������

(�) ��� �����(�) ��� �����, ��� ����(�) ��� ���, ��� ������(�) ��� �����, ��� �������, ��� ����(�) ��� �����, ��� ����, ��� ������

GO ON TO THE NEXT PAGE.

73

Page 26: Official LSAT PrepTestdocshare02.docshare.tips/files/27057/270577487.pdfLSAT® Writing Sample Topic Directions: The scenario presented below describes two choices, either one of which

3 -26- 3 3 3 3��������� ��–��

� ������� �� ������� � ��������’� ����� ��� ����� ��������—������� �, ������� �, ��� ������� �. ���� �� ��� �������� �� �� �� �������� �� ��� �� ���� �� ���� ����� �� �������—������, �������, �����, �����������, ��� ������—������� �� ��� ��������� �����������

�������� � ��� � ������ ���� ��� ���� �� ������ �� ������.

�������� � ��� � ���� ���� ������� ��� ����� �� ������� �� ������.

������� � ���� ���� �����������.�� � ������� ��� ������, ���� ������� ���� ���� ���� ����� ��� ������ ���� �����������.

�� � ������� ��� ������, ���� ������� ���� ���� ���� �������.

��. ����� ��� �� ��� ��������� ����� �� � �������� ��� �������� ���� �� ��� ����� �� ������� �� ���� �� ��� ���������

(�) ������� �� ������, ������������ �� �������, �����, ������������� �� �������, �����������, ������

(�) ������� �� �������, ������������ �� �������, ������������������ �� �������, �����������, ������

(�) ������� �� �������, ������������� �� �����, �����������, ������������� �� �����, �����������

(�) ������� �� ������������ �� �������, ������������������ �� ������, �������, �����������

(�) ������� �� ������������������ �� ������, ������������ �� ������, �����

GO ON TO THE NEXT PAGE.

73

Page 27: Official LSAT PrepTestdocshare02.docshare.tips/files/27057/270577487.pdfLSAT® Writing Sample Topic Directions: The scenario presented below describes two choices, either one of which

3 3 3 3 -27- 3 ��. �� ������ ��� �� ������� �, ����� ��� �� ��� ��������� ����

�� �����

(�) ������ ��� �� ������� �.(�) ������� ��� �� ������� �.(�) ����� ��� �� ������� �.(�) ������ ��� �� ������� �.(�) ������ ��� �� ������� �.

��. �� ������ ��� �� ������� �, ����� ��� �� ��� ��������� ����� �� � �������� ��� �������� ���� �� ��� ����� �� ������� �� ������� ��

(�) �������, ������(�) �������, �����������(�) �������, �����������, ������(�) �������, �����, ������(�) �������, �����, �����������, ������

��. ����� ��� �� ��� ��������� ������ �� � �������� ��� �������� ���� �� ��� ����� �� ������� �� ������� ��

(�) ������, �����(�) �������, ������(�) �������, �����, �����������(�) �������, �����, ������(�) �������, �����, �����������, ������

��. ����� ��� �� ��� ��������� ������ �� �����

(�) ������ ��� ����� ��� ��� ���� ����� �� ������� �� ������� �.

(�) ������� ��� ������ ��� ��� ���� ����� �� ������� �� ������� �.

(�) ������, �������, ��� ����� ��� ��� ���� ����� �� ������� �� ������� �.

(�) �������, �����, ��� ����������� ��� ��� ���� ����� �� ������� �� ������� �.

(�) �������, �����������, ��� ������ ��� ��� ���� ����� �� ������� �� ������� �.

S T O PIF YOU FINISH BEFORE TIME IS CALLED, YOU MAY CHECK YOUR WORK ON THIS SECTION ONLY.

DO NOT WORK ON ANY OTHER SECTION IN THE TEST.

73

Page 28: Official LSAT PrepTestdocshare02.docshare.tips/files/27057/270577487.pdfLSAT® Writing Sample Topic Directions: The scenario presented below describes two choices, either one of which

4 -28- 4 4 4 4 4SECTION IV

Time—35 minutes26 Questions

Directions: The questions in this section are based on the reasoning contained in brief statements or passages. For some questions, more than one of the choices could conceivably answer the question. However, you are to choose the best answer; that is, the response that most accurately and completely answers the question. You should not make assumptions that are bycommonsense standards implausible, superfluous, or incompatible with the passage. After you have chosen the best answer, blacken the corresponding space on your answer sheet.

�. �� �� ����������, ��� ������ ���� ����� �� ����� ������� �� ������ ��� ���� ����. ���� �� ��� ������ ���� ����� ��������� ���� ��� ������, ��� ���� ����� ������������ �������� ���� ��� ��� ������ ������� ������ ������ ���� ��� ���� �� ��� �������. ���� ������ ������ ������ ����, ��� ���� ���� ���� �� ������ �����������. �������, ����, ��������� ���������� ���� ���’� ������� �� ����� ������.

����� ��� �� ��� ���������, �� ����, ���� ���������� ��� ���������� ����� ������

(�) ��� ��� ������ ���� �������� �������� �� ������ ��� ����� ���� ������ ���� ������ �� ��� ����� ���� ��� ���� ����� ���������, �������� ���� ������ ��� ����� �� �� �� ��� ����� ���� �������� ���������.

(�) ������� ������� ���� �������� ���� ��� ���������� ��� �������� ���� � ���������, ������ ����� �� ������.

(�) ��������� �� �������� �������� �� � �����, ������� ������ �� �������� �������� �� � ������.

(�) ��� ���� ������ ��� ���� ���������� ����� ��������� ���� ����� � ���� ����� �� ����� ������ ������� ������� ���������, ��� ���� ����� �������� �� ����������� ������� ��� ������ �������.

(�) ���� �������� ��� ������ ���� ������ �������� ���� ������ ����������� ������� ��� ������ �������, ����� ������ ������� ��� ����������� ���� ������������.

�. ��������� �� � ������� ��������� ��� ��������� ��� ������� �� ��� ������ �� ����� � ����� ����� �� ��� ������’� ����. �������� ��� ����� ����� ��� ������� ��� ���� �����, ��� ���� �� �������� �� �������� �� � �������� �� � ������ ��� ������� ��������� ������� ��� �������� ����. ��������� ����� ���� ��� ����� ����� ����� ��� �������� ��������� ��� ����� ������ ��� ���������’� �������� ��� �������� ��������.

����� ��� �� ��� ��������� ����������, �� �����, ���� ����� �� ������� ��� ��������� �� ��� ��������� ������� �� �������� ��� ������

(�) ����� �������� ������ ������ �� ���� �� �������� �������� ��������� ���� ��� ���� ����� ��� ���� �����.

(�) ����������� ���� ���� �� �������� ����������� ����� �� ��������� �� �������� ������������� ������ �������� ����� ���������� ��� ��� ������ �����.

(�) ������� ��� ������ �� ��������� ������ ���������� �� ����� ��������� ���� ��� ����� �� ������ ���� ��� ������ �� ��� ���� ����������� �� ��� ������������� ����� ��� ���������.

(�) ��� ����� �� ����� �� ���’� ��� �������� �� ����������� �� ��� �������� ��� ���������� ��������� �� ��� ��������� �� �����.

(�) �� �������� �����������, ��� �������� ��� ������ ����������� ������ �������� ������������ ���� ��������� ������ �� �������� �����.

GO ON TO THE NEXT PAGE.

73

Page 29: Official LSAT PrepTestdocshare02.docshare.tips/files/27057/270577487.pdfLSAT® Writing Sample Topic Directions: The scenario presented below describes two choices, either one of which

4 4 4 4 4 -29- 4�. ������ ��������� �������, ����� ��� �������� �� �����

����-������-������� ����������� ���������, �� ��� �� �� �����������. ���� ���������� ������� ���� ����� ���� ������ ��� ���� ������������ ���� ����� ������� �������, �� �������, �� ���� ���� ������� �� ����� ��� ���� ������, �� ����, ���� ����.

��� ��������� �� �����’� �������� �� ���� ���������� �� ��������� �� ��� ������� ���� ��� ��������

(�) ����� ��� ������� ���� ����� ��� �� ����� �������� ������ �������� �� ����� ��������� ������� ����� ���� �������� ����-������-������� ����������� ���������

(�) ����� �� ����������� ������� ��� �������� ������ �� ����� �� ���� ������ ��� ��� �������� �� ����� ����� �� ��������� ������������� �� ���� ������ ����������

(�) ����� �� �������� ��� ������������� �� ��������� ������� ���� ��� ��� ������������ �������� �� ����� ����-������-������� ����������� ���������

(�) ������ �� �������� ��������� ��� ����������� �� ������ �� ��������� ������ ���������� ������ ����, �� ���������, ����� �� ���������� �� ���������

(�) ��������� ��� ����������� ���� ������ ��� ������������ ��� ��������� ������� ����� ���� ���� �� ��� ���, �� �������, ���� ������ ��� �� ���

�. ������������� ���� ���� ������ ���� �������� ������������� �� ��� ���� ��������� ���� ������� �� �� �� ��������� ��� ���� ���������. ��� ������ ���� ���� ��� ������. ������� ���������—�.�., ���������� ���� � ���-������ ����� ��� ��� ���� ��������� ��������� ��� ��������� ���� ������� ��� ���� �� �������� ������� �� ���� ���� ��� ����� �� ���� ���� �� ��� ����—����� ����� �� ��������.

��� ��������� ��������� �������� �� ��� ������������ �� ���� �� ���������� �� ��������� �� �������� ��

(�) ����������� � ��������� ���� ����������� ��� ��� �� ����� ��� �������� �� �����

(�) ����������� ��� ����� �� ��� ��������(�) ���������� �� ��������� �������� �����

���������� �� ������� �� �� ��������� �����(�) �������� ���� ��� �������� �� ����� �� � �����

�������(�) ���������� ���� � �������� ����� �� �

���������� �� �������� �� ������ �� ���� ����������

�. ����� ������������ �� ������������� �� ��� �������� �� ���� �� �����, ������������ ���� ��� ������� ��� �������� �� ����� ��������� �������� �������. ��� ����� �� �� ���� �� �����, ������� ���������� ������ ���� �� ������� ���� ������, �������, ��� �� ������ �� ���� ���� ��� �������.

����� ��� �� ��� ��������� ���������� ���� ���������� ��������� ��� ���������� ����� �� ��� ���������

(�) ������������ ���� ��� ������� ���� ��� �������� ������� �������� �� �����.

(�) ����� ��� ������� ���������� ������ ���� ���� �� ������ �� ����� �� �� �� ���� �� �����.

(�) ��� �������� �� ���� �� ����� ������� ���� ������������.

(�) ����� ��� ���� ���������� ������ ���� ��� �� ������ �� ���� ���� ��� �������.

(�) ��� ������� ���� ������������� ��� ������������ ���� ���� ���� ��� ���������� �����.

�. ���������� �������� ���� ������ ������ ��������� ��� ������� ������� ���� � ����� �� ����� ����, �������������, �������� ��� �������’� ���� �� ���������� ������������, � ����-���� �������. �� ����� �������� ���� ������� ���� ���� ����� �� �������� �������� ����. ����� � ���� ���� �������� ��� ������ �� ��� ���� ����� ��� ��� ������ ���������, ����� �������� ������ ���� ���� ��� ���� �� �������� �� ��� ���� ���� ����� �� �������� �������� ����.

����� ��� �� ��� ��������� ����� �� ���� ������ �� ���� �� ����� �� �������� ��� ���������’� ���������

(�) ��� ����� �� ��� ����� �� ����� ���� �������� ��� ���� �� ���������� �������������

(�) ��� ��� ������ ������ ��������� ������� ���� ����� ���� �������� ��� ���� �� ���������� �������������

(�) �� ��� ��� ���� ���� ��������� ���� ��� ���� ���� ����� �� �������� �������� �����

(�) ��� ���� ��� ��� ���� ���� ����� �� �������� �������� ���� ���� �� ����

(�) �� ���� ������ ���� ��� ��� ���� ������ ��� �������� ���� ���� �� ����������� ���� ��� ����� ������

GO ON TO THE NEXT PAGE.

73

Page 30: Official LSAT PrepTestdocshare02.docshare.tips/files/27057/270577487.pdfLSAT® Writing Sample Topic Directions: The scenario presented below describes two choices, either one of which

4 -30- 4 4 4 4 4�. ������� ��� ��������� ����� �� ��� “���� �� � ����”

���������� �������� ��������� ���� �������� ��������. ��� ����’� ������� ����—��� ������ ����� ��������—�� ������� �� � ����������� ������� ����. ��� ������� �� �� ��� ���� ��� ������ �� ��� ����, �� ������ ������� ��� ������� �� � ����� ��������. �� ������� �� � ���������� ����� �������� �� ��� ��� ������, ����� �� �������� �� � ������� ��������� �������� ����. �� ���������� ������ �������� ��� ����� ��� ���� ���� �����.

��� ������’� ��������� ���� ������� �������� �� ����� ��� �� ��� ��������� �����������

(�) � ����� �������� ���� �� ������� �� � �������� ���� ������, �� ��������, �� ������� �� �� �������� ����.

(�) � ���� ����� �� ���� ����� ��������� �� �� �� �� ���� ������ ��������.

(�) � ����� �������� ���� �� �������� ������� ������ �� ������� �� � ����������� ����.

(�) ��� �������� ���� �� � ���� ������ �� �������� �� � ��� ���� ����������� ��� ����’� ����� ���������.

(�) ��� ������� �� � ����� �������� �� �� ��������� ������ �������� ��� �� ���� � ���� ���� �����.

�. �������� ������ � ������’� ����������� ���� ���� ��������� ��������-������� �������� �� ��� ����. �� � ������ �����, ����������� ����� ���� ���� ��������, �������������� �� ��������-������� ��������, ��� ����� �������� ��������� ��� ���� ��������, �������� �������������� �� ����������� ��� ������������� ������ �� ����� ����� ��� ����������� ���� �������� � ���� ������������� �� ������.

����� ��� �� ��� ��������� ��� ���� ���������� �� ��������� �� ��� ����� �� ��� �����������’ ���������

(�) ��������-������� �������� ��� ��� ��� ������� ������ �� �������� ����� �� �����������.

(�) �������� ���� ��� ������ �������� �������������� �� �����������.

(�) ������-������� �������� �������� �� � ������ ���� ���� ��������-������� ��������.

(�) ������ �� ����������� ��������� ��� ���� �� ����� ��������-������� �������� ��������.

(�) ���� ���� �������� ���� �������������� �� ������-������� �������� ���� �������� ���� �������������� �� ��������-������� ��������.

�. ��������’� ��������� ���� ��� ������ ���������� ����� �������� �� ��� �������� �� ��� ���� ������, � ������������� ��������� �����, �� �������� �� �� �������. ��������’� ���������� �������� ���� �� �����’� ������� ������� ������. �������, ����� �� �������� ���� ����� ��� � ������ �� � ���������� ������, ��� �� �������� ���� �� ��� � ������ �� ��� ��������.

��� ��������� ���� ����� �� �� �������� ���� ����� ��� � ������ �� ��� �������� ������� �� ��� �������� �� ����� ��� �� ��� ��������� �����

(�) �� �� � ������� ����, ���� �������� ���� ��� ����� ��������, ���������� ��� ������� �� ��������’� ���������.

(�) �� �� ���� �� ������� ��� ����� ���� ����� ��� � ������ �� � ���������� ������.

(�) �� �� ���� �� ������� ��������’� ���������� �� ����������� ��������’� �����������.

(�) �� �� �������� �� ���� ����� �� ��������’� ���������� �� ����������� ��� ����������� �� ��������’� ��������.

(�) �� �� �������� �� ���� ���� ���� �����’� ������ �� ������� ������ ������� �����������.

��. ������ ������� ������������ �� ����� ���� ���� ����� �� ������ ������� �� ������ ���� ���� ��� ������ �������, � ��� ���� ��� ������ ���� ���� ���� ����� ������. ���� � ��� ��� ����� �� ���� ��� ���� ����. ��� ���� ���� ���� ��� ����� �� ������ �� �� ���� ���� ��� ������ ���� ������, ������� ��� ����� �� �� ��� ������ ������.

��� ������ ������� �����������’� �������� �� ���� ���������� �� ��� ��������� ���� ��

(�) ������� ��� ����������� ���� ���� �������� �� ��� ������ ���� ����� ��� ����� �� ��� ���� ���� �� ��� ����

(�) ����� ��� ������� ���� ��� ����� �� ���� ����� ������� ������ ���� ��� ��� ������ ������� ���� �� �� ��� ���� ��� �� ��� ��� ������ �������

(�) ����� ��� ������� ���� ������� ��� ����� �� ����������� ��� ������ ������� �� �� ��� ���� ������ ������� ���������

(�) ������� ��� ����������� ���� ��� ������ �� ���� ������� �� ��� ���� �� ��� ��� ������ ������� �� ������� ���� ��� ������ �� ���� ������� �� ��� ����� ��

(�) ������� ��� ����������� ���� �� ��� ����� �� ��� ���� ���� ����� �� ����� ���� ���� ������ ���� �� ���� ���� �� ��� ��� ��� ������ �������

GO ON TO THE NEXT PAGE.

73

Page 31: Official LSAT PrepTestdocshare02.docshare.tips/files/27057/270577487.pdfLSAT® Writing Sample Topic Directions: The scenario presented below describes two choices, either one of which

4 4 4 4 4 -31- 4 ��. �������� ���� �������’� ������ ����� ���� �������

����������� ���������� ������ �� ���� ������� �� ��� ������ ��������� ���. ���� ���� ����� ���� ���� ������� �� �� ��� �� ��� ������ ���� ����� �� �� ���� �� ������� �� �����. ��� ���� �� �� ������������� ��������. ���� ������� �� � ��������� ������� ������� ��� ���� ������� �� � ��-����-��� ������� “���’� �����, ����, ���������� �� �� ����� ����� ��� ���’� ���� �� ���� �������� ���.” ���� ������� ����� �� ������ �� ����� �����������.

����� ��� �� ��� ��������� ���� ���������� ��������� ��� ������� ���������� ����� �� ��� �������’� ���������

(�) ���� ������ ����� ���� ��� ������-����� ������� �� �� ��� �� ��� ������ ���� ����� �� �� ���� �� ������� �� ���.

(�) �� ����� �� ������������� �� �������� ������� �� ��� ������ ��������� ���� ������� ��� ������ ����� ���� ��� ������� ���������� ��� � ���� ����.

(�) �� �� ������� ��� ���� �� ��� ������ ���������, ��� ������ ����� ���� ���������� ������� ����������� ����������.

(�) �� �������� �� ���� ��� ������ �����������, � ������� ������ ������ ��� ���� ��������� ���� ��� ����������� ��� �� ���������� ��� �� �������� ��� ����������.

(�) ��������� �������� ������ ������ ����� ������� �� ���� ����� ��� ���������� ���� �� ���������� �� ���� ����� ����.

��. ���������, ���� ����� ���� ���� ���� ���� ���� �� ��� �������� ����. �� ����������, ����� ���� ���� ��������� ����. ��� ��� ����� ���� ����, ���� �� ��� ��������� ���� ���� ��� �������� ��� ���������. ������, ���� ��������� ���� ���� ���� ��������� �� ������������ �����. �������, ��� �� ��� ��� ����� ��������� �� ����� ��������� �� � �������� ������� ��� ���������, �������� �������-���� �������.

����� ��� �� ��� ��������� �� ���� �������� ��������� �� ��� ����������� ������

(�) ���� ���� ����� ���� ���� ��������� ���� ���� ���� ��� �������� ��� ��������� ���� �� ��� �������� ����.

(�) ��� ����-������� �������� ���� ���� ��� � �������� ���� ��� �������� ��� ���������.

(�) ����� �� ��������� �������� ��� ��������� ���� ����� ���� ���� ���� �� �������� �����.

(�) ���� �� ��� ��������� ��������� ���� ���� ���� ���� ��� �������� ��� ��������� ���� ��������� �� ������������ �����.

(�) �������-���� ������� ���� ���� ������ ���� ���� ���� ��� ��� �������� ������� ��� ������������ �����.

��. �� ����� ����, ���������� �������� ������� �� ��� ���������� �� ����. ������� �� � ������� �������� ���� ����� ����� ���� ��� �� ��� ����������� ��������� �� ��������. �� ��� ������� �� ��� ������� ���������� ���� ���� ���� �������� ���� ��� ���������� ���������� ��������.

��� �������� ������ �� ��� ���������� ����

(�) ���� ��� �� ������� �� ��� ���������� ����� �� ����

(�) ��� ������� �� ��� ������� ���������� �� ���������� ������� �� ��������

(�) ������� ������ �� �������� ����� �� ��� ������� �� ���� �����

(�) ��� ������� ���� ��� ���������� �������� ��� ���� ������� �� ����������� ���������

(�) ������� �� �����’� ���������� ���� ��� ���� ����� �� � ������ �� �������� �� ����������� ���������

��. ����������������� ��������� ���� �������� ������ �� ���� ����������� �� ������� ������������� ��������. ��� ��� ���� �� ����� �������� �� ��� ��������� �� �������� ������, ��� ����� ������� ���� ��� ������ ���������’ ��������� ����� ��� ���� �� �����. ������� ����� �� ��������, �������, ����� ������� ���� ����, ��� �� � ������ ��������� ����� ������� ����.

��� ����������������’� ����������, �� ����, ���� �������� ������� ����� ��� �� ��� ����������

(�) ��� ���� �� ��������� ���� ������� ������ ��� �� ��������� �� ��� ����� �� �������� ������ ��� ������ �� ��������� �������� ����� �� ������� �� � ������.

(�) ������� ����� �� �������� ����� �������� ���������’ ��������� �� ��� ����� �� ������������� �������� �� ����� ��������� ���� ������� �����������.

(�) ��������� ����� �������� ���� ��������, �� �������, �� ��� ���� �� ��� ������������� �������� �� ����� ��������� ���� ������� ����������� ���� ����� ��������� �� ��� ����� �� ��������.

(�) ��� ���� ���� ���������� �� ���� ��������� ���� ���� ��� �������� ��� ���� �� ����� �� ��� ���� �� ��������.

(�) ��������� ���� �������� ������ �� ���� ���� �� ������� ���� �� ��������� ���� ���� ������������� �� ��� ���� �� ���� ��������� ���� �������� ��� ���� �� �����.

GO ON TO THE NEXT PAGE.

73

Page 32: Official LSAT PrepTestdocshare02.docshare.tips/files/27057/270577487.pdfLSAT® Writing Sample Topic Directions: The scenario presented below describes two choices, either one of which

4 -32- 4 4 4 4 4 ��. ����’� ������� ����������� ��� �� ���������� �������

���� ���� �� ��������. ��� ������ �� ����� ����������� ��� ������� ���� �� ��� �����, ����� ���� ��� ������� �� ��������� �� ������� ������� ��������� ��� ����� ��������. ������������, ��� ��������� ����������� ��� ���� ������ �� ������ ������� �� ����� ����� ��� ����� �������� ��� ������� ���� �� ����� ������� ��� ����� ��������.

����� ��� �� ��� ���������, �� ����, ���� ����� �� ������� ��� ���������� �����

(�) ��� ����� �������� ��� �������� ���� ������ ������ ���� ����� ���� ���� ��� ����������� �������� ��� ��.

(�) ��� ����� �������� ������� �� ������ �� ����� ����’� ������� �����������.

(�) ��� ������ ���� �� ��� ��� ����� �������� ���� ��� ������� ��� ������ ������� ���� ���� �� ��������� ������.

(�) ��� ����� �������� ��� ����� �� ���� ���� ��������� ���� ����’� ������� ����������� ���.

(�) ����������� �� ��� ����� �������� �� � ������� �� ��� ���� ������������� �� ��� ����� ���������� �� ����’� ������� ����������� ���� ���.

��. ������ �� � ������ ����� �� ���� ����� ��������. �� ������� ����� ����� ��� ������� �������� ������, ���� ������ ��� ����� ����� ����� ��������. ��� ���� ������ ��� ���� ����� �����, �� ����, �� �������� �� ��������.

����� ��� �� ��� ��������� �� ���� �������� ��������� �� ��� ����������� ������

(�) ��� �� ����� ���� ������, ������ ����� ����� �������� ��� �� ����� ���� �������� �� ����� ����� ������ ������ �� �� �������.

(�) ���� ������ ���� ���� ����� �������� ��� ����� ����� ����� �������� �� �������� ����� ������ ������.

(�) ���� ������ ��� �� ��� �������� ��������� ���� ������ ������ ������ �� � ������.

(�) �������� �� �������� ��� �������� ����� ���’� ����� ��������.

(�) ��� �� ����� ���� ������, �������� �� �������� ��� ����� ����� ������ ������ �� �� �������.

��. � �������� ����������� ��� ���� ����� ������� ��� ������ �� ���� �� ��� ���������� �� ������ ��� ��� ��������� �� � ������� ������� ����� ����� ������’ �����������. �������, ��� ���� ������ �������� ���� ��� ����� ���� �������, ����� �� ������ ����� ����� ��� ����� ������� �� ���� �� ��� ���, ����� ��� ������� ���� ���� �������������� �� ���� ����� ��� ����������.

����� ��� �� ��� ��������� ����������, �� ����, ���� ������� ��� ���������

(�) �� ������ ����� ����� ��� ���� �������������� �� ���� ��� ���������� ��� ������ �� ��� ���� �� ��� ���, ��� ��������� �� ��� ������� �� ���� �� ����, �� �������, �� �� �� �� ������ ����� ����� ��� ����� ������� �� ���� �� ��� ���.

(�) �� ��� ������� ������ ������ ������ �� ������ �� ����� ����� ��� ����� ������� �� ���� �� ��� ���, ���� ��� ���� �� �������� ��� ����� �� ��� �������.

(�) �� ���� �� ��� ������ ����� ����� ��� ����� ������� �� ���� �� ��� ��� ��� ������ ����� ��� ���������, ��� ��������� �� ��� ������� �� �� ����� �� ���� �� �� �� �������� ����.

(�) �� ���� �������������� �� ���� ��������� ���������� �� � ����’� ��� ��� ���������� ���� � ���� ��������� �� ��� ������� ����� ���� ����’� ����������, ���� �� �� �������� ���� ��� �� ���� �� ����� ���������� ���� �������� ����������� �� ��� �������.

(�) �� ������ �� ����� ����� ��� ���� �������������� �� ���� ��� ����������, ����� ��� ��������� ���� ���� �������������� �� ����� ����� �� ��������� ���� ��� ���� ������ �� ���������� �������� �� ��� �������.

GO ON TO THE NEXT PAGE.

73

Page 33: Official LSAT PrepTestdocshare02.docshare.tips/files/27057/270577487.pdfLSAT® Writing Sample Topic Directions: The scenario presented below describes two choices, either one of which

4 4 4 4 4 -33- 4 ��. �� ��� ���� ������ ����� ��� ���� �� ���� �� ��� ������.

��� ������� � ��� ������ �� ���� ���� ����� ���� ������. ����� ���� ��� ���� �� ������ �� ����, �� ���� �������� ���� �� ��� ������ ������ ��� ���� ����.

��� ������ ������� �� ��������� �� ��� �������� ����� �� ���� ������� �� ���� �� ����� ��� �� ��� ��������� ����������

(�) ����� ��� �������� ������������ ��� ��� ��� ���� ��� ����� �� �� ����� �� ��� ������� �������� ��� ��� ����������� �� ������. �������, ����� ��� ��������� � ��� ������ �� �� ����� �� ��� ������� ��������. �� ����� ��� �� ������ �� ��� ����� ���� ����� ��� ��� ��� �������.

(�) ����� ��� ��������� ��� ������ �� �� ����� �� ��� ������� ��������. ����� ��� �������� ������������ ��� ��� ��� ���� ��� ����� �� �� ����� �� ���� ������� ��� ��� ����������� �� ������. �����, ����� ��� �������� �� ������ �� ��� ����� ���� ����� ��� ��� ��� ������� �� ���� ����� �� ��� �������.

(�) �� �������, ����� ��� � ��� ������ �� �� ����� �� ��� ������� ��������. ����� ��� �������� ������������ ��� ��� ��� ���� ��� ����� �� �� ����� �� ���� ������� ��� ��� ����������� �� ������. �� ����� ��� �������� ������ �� ��� ����� ��� ��� ��� ��� ������� �� ���� ����� �� ��� �������.

(�) ����� ��� ��������� ������� ������ �� ��� ������� �������� ��� ��� ����������� �� ������. ��� ����� ��� ������ ��� ������ �� �� ����� �� ���� �������. �� ����� ���� �������� �� �� ������ �� ��� ���� ����� �� ��� ������� �������� ���� ��� ����������.

(�) ����� ������� ��� ������ �� ���� ����� �� ��� ������� ��������. ����� ����� ��� �������� ������������ ��� ������ ����� �� ���� ������� ��� ��� �������� �� ������, ����� ��� �������� ���� � ������� �� ��� ������������.

��. ����� ������ ������� ������� �� ������ ���������, �������, ��� ��� ��� ������ �� �������. ���� ��������� �� �������� ������� �� ��������’� ������, ������� ������ ���� ���������� �� ���� ��� ����� ���� ����� ��� ������������. �� ���� �� ���� �� ��� �� ������� ���� ���� �� ���������� ��� ��������� ���� ������ ��� �������� �� ������� ���� ������ ������� ��� ��������.

����� ��� �� ��� ��������� �� �� ���������� �� ����� ��� �������� ��������

(�) ���������� ������ �� ���� �� ����� ��������’� �������� ���� ������ ���� �� ������� ���� ��� ������ ��� �� ������ ��� ���� ����� ���� �� �� ���� �����������.

(�) �������� ������ �� ������� ���� ������ ������� ��� �������� ���� ���� ������� ��� ���� �� ���������� ���� ������� ���� ����������� ������� ����� ��� ������.

(�) �������� ����� ��� ������ �� ������ ���� ���������, ���� �� ���� �������� �� ����� �� �� �������� ������ �� ���� �� ������ �� �����.

(�) �� ������ ����������� ������� ���� ��� ������ ��� �� ������, ���� ���������, �������, ��� ��� ���� �� ����������.

(�) ��� ������� ���������� �� ���� �������� �� ��������� ��� ������� ����� ���� �������� �����������’ ��������� �� ������� ���� ������� ��� �� ������.

��. �� � ������ ����� �� ������ ��������, ����� ��� ��������� ���������� ������������� �� ��� ����� ����� �� ��� ����� ����� ��� ������ ���� ��������� ��� ������� ������ �� ��� ������� ��������� �� ����� �����. ���������, ����� ��������� ������ ����� ����� �� � ����������������, ��� ���� ����������� ����� ����� �� �� ����� ���� ������� �� ������-������� ����� �����. ���� ��������� ������� ���� ������� �� ������-������� ����� ����� �� � ����� �� ����-���� ����� ������ ��������� ���� �������.

����� ��� �� ��� ���������, �� ����, ���� ����������� ��� ���������

(�) ��� ���������������� ���� ����� ���� � ������� �� ������-������� ����� ���� ���� ������ ����������� ����� �����.

(�) ������ �������� ������� � ���� ������� �� �������� �������� ������ �� ����� �����.

(�) ��������� �� ��� ���� ���������������� ���� ����� ���� ������-������� �� ���������� ������� ����� �����.

(�) ������� ���� ������� �� ������-������� ����� ����� �� ��� ���� �������� ������ �� ��������� �� ��� �����.

(�) ����� ����� ��� ������ ������ ������ �� ���� ��������� ������� ����� ��������� ����������.

GO ON TO THE NEXT PAGE.

73

Page 34: Official LSAT PrepTestdocshare02.docshare.tips/files/27057/270577487.pdfLSAT® Writing Sample Topic Directions: The scenario presented below describes two choices, either one of which

4 -34- 4 4 4 4 4 ��. ��� ���� ����� ������ ��� ���� ������� ��� ����� �����

��� ���� ���� �����. ���� ���� ���� ����� ������� �� ���� ���� ����� ������. �� �� ��� ���� ���� ������ ������ �� ��� � ����� ����, �� �������� ���� ��� ������� ���� ���� ����� ������.

��� ��������� �� ����� ��� �� ��� ��������� ��������� �� ���� ������� �� ���� �� ��� �������� ������

(�) ��� ���� ���� ��� ����� ������ ��� ���� ����� ��� ���� ��� �������. ���� ������� ��� ���� �����. �� �� ��� ���� ��� ��� ����� ������ ���� �� � ������, �� ���� �������� �� ���� �����.

(�) ���� ������� ��� ���� �����. ��� ����� ������ ��� ����� ����� ��� ���� ����� ���� ���� ��� �������. �� �� ��� ����� ������ ��� ���� ����� � ����� ���, �� ��� �������� � ������.

(�) ��� ��� ���� ��� ����� ������ ��� ���� ����� ���� ���� ���� ��� �������. ���� ������� ��� ���� �����. �� ��� ��� ��� ����� ������ ���� ���� ���� �� ��� � ���� ���� �������� �� ���� �����.

(�) ����� ��� ��� ����� ������ ��� ���� ����� ��� ���� � ���� �� � ������. ���� ������� ��� ���� �����. �� �� ��� ���� ��� ��� ����� ������ ���� �� ��� � ������, �� ���� �������� ��� �� ���� �����.

(�) ��� ����� ������ ��� ����� ����� ��� ���� ����� ���� ���� ��� �������. ���� ������� ��� ���� �����. �� ��� ���� ��� ��� ����� ������ ���� ���� �������� �� ���� ����� �� �� �� ��� � ����.

��. ����������� ����� �� ���� � ������� ��������� �� �������� ����������� ��������� ������ �� ������ ���� �� ���� �� �������� ����������� ��������� ������ �� ����. ���, ����� ��� ������� ����������� ����� ����� ����������� ��� �����-����� ��������, ����� �� ��� �����-����� ���� ��������� ���� ���� ����� �� ��� �����-����� ������.

����� ��� �� ��� ���������, �� ����, ���� ����� �� ������� ��� �������� ����������� ��������� ������

(�) ����� ��� �������� ���� ��������� ������ �� ������� ������ ��� ��� �����’� ��� ����� �� ������ �� ���� �� ��� ����� ������ �� ������.

(�) ��� ������� ������� ����� �� ������� ��� ��� ������� �� ������ ���� ���� ��� �� ���� ��� ��������� �� ���� ������ ������.

(�) �������� �� ������� ���� �� �������� ��� �����’� ��� ����� �� ����� �������� ���� ���������� ���� ����� ������ �� ����� ��������.

(�) ������������ ���������� ��� ������ ������ ���� ����������� � ����� �������� �� ����� �� ������ ��������.

(�) ��������� ���� �� �������� ����� ������ �� ������, ��� �������� ��������� ����� �� ���� �� ����� ���������.

��. �������� �� ������ ������������� ������ �� ��� �����, ��� ���� ������������ ��� ���� ��������. �������, �� ������ ���� ��� ������������ ������, ��� ���� ���� ��� ��������.

���������� ��� ������ ����� �� ��������� ����� ���’� ��������, ������ �������� ����� ��� �������� ���� ��� ������ ���� ���� ��� �� �������� ����� ������ �������� �� �������.

��� ���������, �� �����, ���� ����� �� ������� ��� ��������� �� ����� ��� �� ��� ��������� ��������� ���������� ��� ��������

(�) �� ������ ������������� ������, ��� ���� ����� ������� ��� ��������, �� ��� ������ ������������ ���.

(�) ������ ����� ������ ��� ������ �� ������������ ���—���� �� �����������—������ ���� ��� ��� �� ������� ��� ���� ��������, ��� �� ������ ����� �� ����� ������� ����� �� ����������� �������������� ������.

(�) ������ �������� ���� �������� ������ �� ��������� �� ���������� ���� �������� �� ������, �� ��� ������ ��� ���� ������ ��� ���� ��������.

(�) ������’� �������� ����� �� ���� �� �� ���� ���� ��������������� ���� ������ ���� ���������, �� ������ ������ ��� ������������ ���.

(�) ������ ��� �� ������� ����� ������� ������ ����� ������ �������� �� �������, �� ��� ������ ��� ������������ ���.

��. �������, � ��������� ������ ������ ������� ��������� ���� �����. ����� ���, � ��������� ������ ������ ������� �� ���������� ���� �� ������������� ����� ��������� ��������� ������, ��� �� ���������� ��� �� ������������� �� ���� ��� ���� �� �� ��� ������ �� �������� ����������� ����� ��� ����������.

��� ��������’� ���������� �� �������� �������� �� ����� ��� �� ��� ��������� �� ��������

(�) ��� ��������� ���� ���� ��������� ���� ����� ��� �������� �����������.

(�) �� �� ���������� ��� ������ �� �������� ����������� ����� ��� ����������, ���� ���� ���������� ���� �� ������������� ����� ��������� ��������� ������.

(�) � ��������� ���� ������ �� ��� ���������� �� ������������� ����� ��������� ��������� ������.

(�) � ��������� ������ ������ �� ��� ���������� �� ������� �� ������ ����������� ����� ��� ����������.

(�) ������� ��������� ���� �����, �� ���������� ���� ��� ���� ������ �� �������� ����������� ����� ��� ����������.

GO ON TO THE NEXT PAGE.

73

Page 35: Official LSAT PrepTestdocshare02.docshare.tips/files/27057/270577487.pdfLSAT® Writing Sample Topic Directions: The scenario presented below describes two choices, either one of which

4 4 4 4 4 -35- 4 ��. ������� �� ��������� ���� ���� ��� �� �����, ��� �����

������ ���� ���� ��� �� �����. ����� ��� ��� ���� ������� ��� ���, ��� ��� ���� ���� ��� ����, ��� �� ������ ��������� ���������.

��� ��������� ����� �� ������ �� ���� ��

(�) ������ ���� � ����������� ������� ��������� ��� ������� ���� ��������� ������ �������

(�) ������� ��� ���������� ���� �� ���� ��� �� �����(�) ����������� �� ��� ����� �� � ������ ��������(�) ����� � ��������� ��������� ��� �������’� ������

������ �� �� � ���������� ���������(�) ����� � ��������� ��������� ��� �������’� �����

��������� �� �� � ���������� ���������

��. ������� ����� ����������� ���������� �� ������� ������ ���� ������ ���� �� ������ ������ ����� �� ��� ���� �� �� ������� ���� ������������. ����� ��������� �������� �� ������������ ��� ������ �������� ����������� ���� ���������, ����� ���� ����������� ���������� ����� ���� ������ �� ������ �� ���� �������� ����������� �� �������.

����� ��� �� ��� ��������� �� �� ���������� ��� ������’� �������� ���������

(�) ��� �� ��� ����� ���� ���� ���� ����������� ���������� �� ������ ������ ����� �� ����� �� ����� ���� ���� ���� ������� ���� ������������ ���� ����� ���� ���� �� �������� �����������.

(�) �������� ����������� ���� ���� ���� ������ �� ��� ��������� �������� �� ������������ �� ��������� ��� ������ �� ������� �� ��� ������ �� ������������ ������� �� ����� ��������� �� ������� ���� ��������.

(�) ����� ���� ���� ���� ����������� ���������� �� ������ ������ ����� ��� ����� ������� ���� ������������ ���� ���� �������� �����������.

(�) ��� �� ����� ���� ���� ���� ����������� ���������� �� ������ ������ ����� �� �� ���� ������ �� ������� ���� ��� ��� �� ������������ �� ����� ���� ��� ��� ����������� ����������.

(�) �� � ����� ��� �� ������� ����� ���� ���� ���� ����������� ���������� �� ������ ������ ����� �� ������ �� ���� �� ����� ���� �������� ����������� �� �������, �� �� ������ �� ���� ���� ������ ���� ������� ��� ��� ���� ������� ��������� ��� ����������� �������� �� ������������ �� ���������.

S T O PIF YOU FINISH BEFORE TIME IS CALLED, YOU MAY CHECK YOUR WORK ON THIS SECTION ONLY.

DO NOT WORK ON ANY OTHER SECTION IN THE TEST.

73

Page 36: Official LSAT PrepTestdocshare02.docshare.tips/files/27057/270577487.pdfLSAT® Writing Sample Topic Directions: The scenario presented below describes two choices, either one of which

36

ACKNOWLEDGMENTS

Acknowledgment is made to the following sources from which material has been adapted for use in this test booklet:

Stephen Jay Gould, “Darwinian Fundamentalism.” ©1997 by Stephen Jay Gould.

Helen Irving, “Little Elves and Mind Control.” ©1991 by The Australian Journal of Media and Culture.

David Lyons, “The New Indian Claims and Original Rights to Land” in Reading Nozick. Edited by Jeffrey Paul. ©1981 by Rowman and Littlefield.

Janet Malcolm, “The Genius of the Glass House.” ©1999 by NYREV.

Susan Milius, “Built for Blurs.” ©2005 by Science Services, Inc.

Robert Nozick, Anarchy, State, and Utopia. ©1974 by Basic Books, Inc.

Page 37: Official LSAT PrepTestdocshare02.docshare.tips/files/27057/270577487.pdfLSAT® Writing Sample Topic Directions: The scenario presented below describes two choices, either one of which

37

Directions:

1. Use the Answer Key on the next page to check youranswers.

2. Use the Scoring Worksheet below to compute yourraw score.

3. Use the Score Conversion Chart to convert your rawscore into the 120-180 scale.

Scoring Worksheet

1. Enter the number of questions you answeredcorrectly in each section.

Number Correct

SECTION I ................. SECTION II ................ SECTION III ............... SECTION IV ...............

2. Enter the sum here: This is your Raw Score.

Reported Raw ScoreScore Lowest Highest

COMPUTING YOUR SCORE

Conversion ChartFor Converting Raw Score to the 120-180 LSAT

Scaled ScoreLSAT Form 4LSN110

180179178177176175174173172171170169168167166165164163162161160159158157156155154153152151150149148147146145144143142141140139138137136135134133132131130129128127126125124123122121120

98970

9695949392919089878685838280797775747270696765636160585654535149474644434139383735343231302927262524232221201918170

101970

96959493929190898886858482817978767473716968666462605957555352504846454342403837363433313029282625242322212019181716

*There is no raw score that will produce this scaled score for this form.

Page 38: Official LSAT PrepTestdocshare02.docshare.tips/files/27057/270577487.pdfLSAT® Writing Sample Topic Directions: The scenario presented below describes two choices, either one of which

38

ANSWER KEY

SECTION IV

1. D2. D3. E4. C5. A6. E7. E

8. D9. D10. E11. B12. A13. B14. C

15. A16. E17. C18. C19. B20. D21. E

22. D23. E24. E25. E26. A

SECTION III

1. B2. C3. B4. E5. E6. D7. A

8. A9. B10. A11. C12. C13. E14. A

15. D16. D17. B18. E19. A20. B21. E

22. A23. C

SECTION II

1. C2. E3. A4. D5. D6. D7. C

8. B9. C10. A11. B12. A13. B14. E

15. B16. A17. B18. E19. B20. E21. E

22. A23. C24. D25. C

SECTION I

1. A2. B3. D4. B5. A6. E7. C

8. B9. A10. D11. C12. E13. E14. B

15. B16. C17. B18. E19. A20. E21. A

22. B23. A24. D25. E26. D27. A

Page 39: Official LSAT PrepTestdocshare02.docshare.tips/files/27057/270577487.pdfLSAT® Writing Sample Topic Directions: The scenario presented below describes two choices, either one of which

LSAT® Writing Sample Topic

Directions: The scenario presented below describes two choices, either one of which can be supported on the basis of the information given.Your essay should consider both choices and argue for one over the other, based on the two specified criteria and the facts provided. There is no “right” or “wrong” choice: a reasonable argument can be made for either.

Scratch PaperDo not write your essay in this space.

©2014 by Law School Admission Council, Inc. All rights reserved.

⭱ ⮝⮕⮔⮙⮥⮝-⮣⮙⮪⮕⮔ ⮓⮟⮝⮠⮑⮞⮩ ⮙⮣ ⮜⮟⮓⮑⮤⮕⮔ ⮙⮞ ⮑ ⮤⮕⮓⮘⮞⮟⮜⮟⮩ ⮠⮑⮢⮛ ⮙⮞ ⮑ ⮣⮠⮑⮢⮣⮕⮜⮩ ⮠⮟⮠⮥⮜⮑⮤⮕⮔ ⮑⮢⮕⮑ ⮟⮥⮤⮣⮙⮔⮕ ⮑ ⮝⮑⮚⮟⮢ ⮓⮙⮤⮩. ⭹⮤ ⮘⮑⮣ ⮘⮑⮔ ⮔⮙⮙⮓⮥⮜⮤⮩ ⮢⮕⮤⮑⮙⮞⮙⮞ ⮕⮝⮠⮜⮟⮩⮕⮕⮣ ⮒⮕⮓⮑⮥⮣⮕ ⮟ ⮤⮘⮕ ⮜⮟⮞ ⮑⮞⮔ ⮕⮨⮠⮕⮞⮣⮙⮦⮕ ⮓⮟⮝⮝⮥⮤⮕ ⮒⮕⮤⮧⮕⮕⮞ ⮤⮘⮕ ⮓⮙⮤⮩ ⮑⮞⮔ ⮧⮟⮢⮛ ⮤⮘⮑⮤ ⮞⮕⮑⮢⮜⮩ ⮑⮜⮜ ⮟ ⮙⮤⮣ ⮕⮝⮠⮜⮟⮩⮕⮕⮣ ⮑⮓⮕. ⭳⮟⮞⮣⮕⮡⮥⮕⮞⮤⮜⮩, ⮤⮘⮕ ⮓⮟⮝⮠⮑⮞⮩ ⮧⮙⮜⮜ ⮙⮝⮠⮜⮕⮝⮕⮞⮤ ⮑ ⮓⮟⮝⮝⮥⮤⮙⮞ ⮑⮣⮣⮙⮣⮤⮑⮞⮓⮕ ⮠⮜⮑⮞. ⭹⮤ ⮝⮥⮣⮤ ⮔⮕⮓⮙⮔⮕ ⮒⮕⮤⮧⮕⮕⮞ ⮟⮠⮕⮢⮑⮤⮙⮞ ⮑ ⮢⮕⮕ ⮒⮥⮣ ⮟⮢ ⮕⮝⮠⮜⮟⮩⮕⮕⮣ ⮑⮞⮔ ⮣⮥⮒⮣⮙⮔⮙⮪⮙⮞ ⮕⮝⮠⮜⮟⮩⮕⮕⮣’ ⮓⮟⮣⮤⮣ ⮟ ⮥⮣⮙⮞ ⮠⮥⮒⮜⮙⮓ ⮤⮢⮑⮞⮣⮠⮟⮢⮤⮑⮤⮙⮟⮞. ⮅⮣⮙⮞ ⮤⮘⮕ ⮑⮓⮤⮣ ⮒⮕⮜⮟⮧, ⮧⮢⮙⮤⮕ ⮑⮞ ⮕⮣⮣⮑⮩ ⮙⮞ ⮧⮘⮙⮓⮘ ⮩⮟⮥ ⮑⮢⮥⮕ ⮟⮢ ⮟⮞⮕ ⮠⮜⮑⮞ ⮟⮦⮕⮢ ⮤⮘⮕ ⮟⮤⮘⮕⮢ ⮒⮑⮣⮕⮔ ⮟⮞ ⮤⮘⮕ ⮟⮜⮜⮟⮧⮙⮞ ⮤⮧⮟ ⮓⮢⮙⮤⮕⮢⮙⮑⭪

● ⮄⮘⮕ ⮓⮟⮝⮠⮑⮞⮩ ⮧⮑⮞⮤⮣ ⮤⮟ ⮝⮙⮞⮙⮝⮙⮪⮕ ⮙⮤⮣ ⮕⮝⮠⮜⮟⮩⮕⮕⮣’ ⮓⮟⮝⮝⮥⮤⮙⮞ ⮕⮨⮠⮕⮞⮣⮕⮣ ⮑⮞⮔ ⮢⮥⮣⮤⮢⮑⮤⮙⮟⮞⮣. ● ⮄⮘⮕ ⮓⮟⮝⮠⮑⮞⮩ ⮧⮑⮞⮤⮣ ⮢⮕⮜⮙⮑⮒⮙⮜⮙⮤⮩ ⮑⮞⮔ ⮜⮕⮨⮙⮒⮙⮜⮙⮤⮩ ⮙⮞ ⮙⮤⮣ ⮕⮝⮠⮜⮟⮩⮕⮕⮣’ ⮧⮟⮢⮛ ⮣⮓⮘⮕⮔⮥⮜⮕⮣.

⮅⮞⮔⮕⮢ ⮤⮘⮕ ⮙⮢⮣⮤ ⮠⮜⮑⮞, ⮤⮘⮕ ⮓⮟⮝⮠⮑⮞⮩ ⮧⮟⮥⮜⮔ ⮜⮕⮑⮣⮕ ⮑ ⮒⮥⮣ ⮑⮞⮔ ⮘⮙⮢⮕ ⮑ ⮔⮢⮙⮦⮕⮢. ⮄⮘⮕ ⮒⮥⮣ ⮧⮟⮥⮜⮔ ⮝⮑⮛⮕ ⮣⮕⮦⮕⮢⮑⮜ ⮔⮑⮙⮜⮩ ⮓⮙⮢⮓⮥⮙⮤⮣ ⮒⮕⮤⮧⮕⮕⮞ ⮤⮘⮕ ⮓⮟⮝⮠⮑⮞⮩’⮣ ⮜⮟⮓⮑⮤⮙⮟⮞ ⮑⮞⮔ ⮑ ⮣⮙⮞⮜⮕ ⮔⮟⮧⮞⮤⮟⮧⮞ ⮣⮤⮟⮠, ⮑⮓⮓⮕⮣⮣⮙⮒⮜⮕ ⮒⮩ ⮠⮥⮒⮜⮙⮓ ⮤⮢⮑⮞⮣⮠⮟⮢⮤⮑⮤⮙⮟⮞ ⮑⮞⮔ ⮓⮜⮟⮣⮕ ⮤⮟ ⮑ ⮜⮑⮢⮕, ⮙⮞⮕⮨⮠⮕⮞⮣⮙⮦⮕ ⮠⮑⮢⮛⮙⮞ ⮑⮢⮑⮕. ⮄⮘⮕ ⮟⮞⮜⮩ ⮢⮙⮔⮕⮢⮣ ⮟⮞ ⮤⮘⮕ ⮒⮥⮣ ⮧⮟⮥⮜⮔ ⮒⮕ ⮤⮘⮕ ⮓⮟⮝⮠⮑⮞⮩’⮣ ⮕⮝⮠⮜⮟⮩⮕⮕⮣. ⮄⮘⮕ ⮒⮥⮣ ⮘⮑⮣ ⮢⮕⮓⮜⮙⮞⮙⮞ ⮣⮕⮑⮤⮣ ⮑⮞⮔ ⮢⮕⮕ ⮇⮙-⭶⮙. ⮄⮘⮕ ⮑⮦⮕⮢⮑⮕ ⮤⮟⮤⮑⮜ ⮓⮟⮝⮝⮥⮤⮕ ⮤⮙⮝⮕ ⮟⮢ ⮑⮞ ⮕⮝⮠⮜⮟⮩⮕⮕ ⮧⮟⮥⮜⮔ ⮒⮕ ⭧⭥ ⮝⮙⮞⮥⮤⮕⮣ ⮕⮑⮓⮘ ⮧⮑⮩. ⭱ ⮒⮢⮕⮑⮛⮔⮟⮧⮞ ⮟ ⮤⮘⮕ ⮒⮥⮣ ⮧⮟⮥⮜⮔ ⮒⮕ ⮔⮙⮣⮢⮥⮠⮤⮙⮦⮕ ⮤⮟ ⮤⮘⮕ ⮓⮟⮝⮠⮑⮞⮩’⮣ ⮟⮠⮕⮢⮑⮤⮙⮟⮞⮣.

⮅⮞⮔⮕⮢ ⮤⮘⮕ ⮣⮕⮓⮟⮞⮔ ⮠⮜⮑⮞, ⮤⮘⮕ ⮓⮟⮝⮠⮑⮞⮩ ⮧⮟⮥⮜⮔ ⮠⮑⮢⮤⮙⮑⮜⮜⮩ ⮢⮕⮙⮝⮒⮥⮢⮣⮕ ⮕⮝⮠⮜⮟⮩⮕⮕⮣’ ⮓⮟⮣⮤ ⮟ ⮥⮣⮙⮞ ⮠⮥⮒⮜⮙⮓ ⮤⮢⮑⮞⮣⮠⮟⮢⮤⮑⮤⮙⮟⮞ ⮤⮟ ⮓⮟⮝⮝⮥⮤⮕ ⮤⮟ ⮧⮟⮢⮛. ⮄⮘⮕ ⮑⮦⮕⮢⮑⮕ ⮣⮑⮦⮙⮞⮣ ⮟⮢ ⮑⮞ ⮕⮝⮠⮜⮟⮩⮕⮕ ⮧⮟⮥⮜⮔ ⮒⮕ ⮑⮒⮟⮥⮤ ⭨⭠ ⮠⮕⮢⮓⮕⮞⮤. ⭽⮟⮣⮤ ⮟ ⮤⮘⮕ ⮕⮝⮠⮜⮟⮩⮕⮕⮣ ⮜⮙⮦⮕ ⮧⮙⮤⮘⮙⮞ ⮧⮑⮜⮛⮙⮞ ⮔⮙⮣⮤⮑⮞⮓⮕ ⮤⮟ ⮑ ⮒⮥⮣ ⮣⮤⮟⮠. ⭽⮟⮣⮤ ⮕⮝⮠⮜⮟⮩⮕⮕⮣ ⮧⮟⮥⮜⮔ ⮘⮑⮦⮕ ⮤⮟ ⮝⮑⮛⮕ ⮟⮞⮕ ⮟⮢ ⮤⮧⮟ ⮤⮢⮑⮞⮣⮕⮢⮣. ⭲⮥⮣⮕⮣ ⮑⮢⮕ ⮣⮓⮘⮕⮔⮥⮜⮕⮔ ⮤⮟ ⮑⮢⮢⮙⮦⮕ ⮕⮦⮕⮢⮩ ⮘⮑⮜ ⮘⮟⮥⮢ ⮑⮤ ⮑ ⮒⮥⮣ ⮣⮘⮕⮜⮤⮕⮢ ⮙⮞ ⮤⮘⮕ ⮤⮕⮓⮘⮞⮟⮜⮟⮩ ⮠⮑⮢⮛. ⭲⮥⮣⮕⮣ ⮑⮢⮕ ⮣⮟⮝⮕⮤⮙⮝⮕⮣ ⮜⮑⮤⮕. ⭾⮟⮞⮕ ⮟ ⮤⮘⮕⮝ ⮘⮑⮦⮕ ⮇⮙-⭶⮙. ⮄⮘⮕ ⮑⮦⮕⮢⮑⮕ ⮤⮟⮤⮑⮜ ⮓⮟⮝⮝⮥⮤⮕ ⮤⮙⮝⮕ ⮟⮢ ⮑⮞ ⮕⮝⮠⮜⮟⮩⮕⮕ ⮧⮟⮥⮜⮔ ⮒⮕ ⭦⭠ ⮝⮙⮞⮥⮤⮕⮣ ⮕⮑⮓⮘ ⮧⮑⮩. ⮇⮀-⮆⭡⭢⭧⭱